You are on page 1of 96

DPP Daily Practice Problems

Solution

Physics

CLASS-XII
NAME- ------------------------------------------------------------

CLASS- ------------------------------------------------------------

BATCH- ------------------------------------------------------------

HAVE CONTROL ¾® HAVE PATIENCE ¾® HAVE CONFIDENCE Þ 100% SUCCESS


PREFACE

This book contains the Daily Practice Problems (DPPs) designed for the aspirants

JEE(Main+Advanced). It is a collection of problems (Physics, Chemistry &

Mathematics in separate booklets) from multiple topics to understand the application

of concepts learned in theory. Each DPP is kind of a timed test with marking scheme

and prescribed time to be spent on each problem. It enables a student to practice time

management while solving a problem.

It covers all the pattern of problems asked in Target exam. Answer Key and Hints &

Solutions are also given for self evaluation. In all, it is a great tool for regular

practice of problems in a systematic manner.

Every effort has been taken to keep this book error free, however any suggestions to

improve are welcome at arth coaching institute.

©Copyright reserved.
All rights reserved. Any photocopying, publishing or reproduction of full or any part of this study material is strictly prohibited. This material belongs to enrolled
13RDLP

student of ARTH only. Any sale/resale of this material is punishable under law, subject to Kota Jurisdiction only.

HAVE CONTROL ¾® HAVE PATIENCE ¾® HAVE CONFIDENCE Þ 100% SUCCESS


DPPs File
(Class-XII)

INDEX
S.No. Topics Page No.

01 DPPs (01 to 82) 001 – 154

02 Answer Key 155 – 159

HAVE CONTROL ¾® HAVE PATIENCE ¾® HAVE CONFIDENCE Þ 100% SUCCESS


HAVE CONTROL ¾® HAVE PATIENCE ¾® HAVE CONFIDENCE Þ 100% SUCCESS
DPP NO. - 1 5. Suppose particle strikes wedge at height ‘S’ after time
1
t. S = 15t – 10 t2 = 15t – 5 t2. During this time
dy dx 2
2. y = ax 2 = c = 2ax distance travelled by particle in horizontal direction =
dt dt
5 3 t. Also wedge has travelled travelled extra distance
2
d2 y  dx  d2 x
= 0 = 2a   + 2ax
dt 2  dt  dt 2

2 2
d2 x  dx  1  c  1 c2
=–       = 
dt 2  dt  x  2ax  x 4a 2 x 3

c2
=  S 15t  5t 2
4a 2 x= =
tan30 1/ 3

3. (D) The extension is spring is x = 2R cos 30° – R = Total distance travelled by wedge in time t = 10 3 t.
= 5 3 t + 3 (15 – 5t2)  t = 2 sec.
 3  1R Alternate Sol.
(by Relative Motion)

15

30°

10 3 5 3

Applying Newton's second law to the bead normal to


circular ring at point B 2u sin 30 2x10 3 1
T = g cos 30 =  = 2 sec.
10 3
N=k  3  1R cos 30° + mg cos 30°
 t = 2 sec.

=
 3  1 mg  3  1 R cos 30° + mg cos 30°
R T
6. It T be the time period ; time to go from O to Q is
12
3 3 mg
N= . T
2 and from M to P is .
6

 v  v0 
4. f  f0  

 v  vs 
A
The displacement is when particle goes from O
 300  2  2
when approaching : fa = 150  
 300  10  to Q, from O to N to Q, from O to N to O to P, and
so on
 300  2  T T T 5T
when receding : fr  150    t= or t =  
 300  10  12 4 6 12
 fa – fr  12 Hence (A).
T T 7T
or t =  
2 12 12

DPPS FILE # 160

HAVE CONTROL ¾® HAVE PATIENCE ¾® HAVE CONFIDENCE Þ 100% SUCCESS


Hence possible time period T is
9. The density of liquid is four times that of cylinder,
12  1 hence in equlibrium postion one fourth of the cylinder
T = 12 s or T = = 2.4 s is submerged.
5
So as the cylinder is released from initial postion, it
12  1 3
or T = s moves by to reach its equlibrium position. The
7 4
upward motion in this time is SHM. Therefore required
A
similarly displacement is when particle goes from 4g 3
2 velocity is vmax = A.  = and A = . Therefore
 4
M to P or M to N to P
Hence the possible time period T is 3
vmax = g
2
6 1
T = 1 × 6 = 6 s or T = s = 1.2 s
5 10. The require time is one fourth of time period of SHM.
Ans. T = 1.2 s, 6s, 2.4s, 12s  

Therefore t = =
2 4 g
7. There is no horizontal force on block A, therefore it
does not move in x-direction, whereas there is net DPP NO. - 2
downward force (mg – N) is acting on it, making its
acceleration along negative y-direction.
Block B moves downward as well as in negative x-
direction. Downward acceleration of A and B will be
equal due to constrain, thus w.r.t. B, A moves in posi-
tive x-direction. 1.

B AN = 3v0 cos2 37º


P0  16 
PB =  v 0  3v 0  
Normal reaction due to C v0  25 
 48 
= 1   = P (73/25) Ans. (B)
Due to the component of normal exterted by C on B,  25  0

it moves in negative x-direction.


1
NA 2. mVm2 = 15 × 10–3
2
B Vm = 0.150 m / s
NC
A = 0.150 m / s
Mg
g
The force acting vertically downward on block B are L q m. = 0.150 m / s
L
mg and NA(normal reaction due to block A). Hence
0.150 0.150
the component of net force on block B along the gL = 3  L= = 1.5 m
100  10 0 .1
inclined surface of B is greater than mg sin. Therefore
the acceleration of 'B' relative to ground directed along at
the inclined surface of 'C' is greater than g sin  3. Angular acceleration () =
r

 d v
Since, at = = constant
dt
 magnitude of  is constant
Also its direction is always constant (perpendicu-
8. lar to the plane of circular motion).

whereas, direction of at changes continuously a t
is not constant.

DPPS FILE # 161

HAVE CONTROL ¾® HAVE PATIENCE ¾® HAVE CONFIDENCE Þ 100% SUCCESS


4. x2 = 4ay 10. (A) p,q (B) q,r (C) q,r (D) s
Differentiating w.r.t. y, we get  1 1 
(A) x  2  sin t  cos t 
 2 2 

x= 2 sin (t – 4 ) is periodic with SHM.
(B) x = sin3 t can not be written
as x = A sin(' t + ) so it is not SHM
but periodic motion.
dy x (C) Linear combination of different periodic function
=
dx 2a is also periodic function.
dy d2 x
 At (2a, a), =1 is not directly proportional to x i.e. this motion
dx dt 2
 hence  = 45° is not SHM
the component of weight along tangential (D) x continuously decreases with time. So x is not
direction is mg sin . periodic function.
g
hence tangential acceleration is g sin  =
2 DPP NO. - 3
5. (C) FBD 1. With respect to the cart, equilibrium position of
the pendulum is shown.
1N T If displaced by small angle  from this position,
2kg T 3kg 8
then it will execute SHM about this equilibrium
fmax = 2 fmax = 6 position, time period of which is given by :
Net force without friction on system is ‘7N’ in right
side so first maximum friction will come on 3 kg
block.
So f2 = 1 N, f 3 = 6 N, T = 2N

6. As point of application of force is not moving,


L
therefore work done by the force is zero. T = 2 g ; geff = g 2  ( 3g )2
eff

9. Wave velocity in string is  geff = 2g  T = 1.0 second

T 40 a
v = = = 20 m/s 2. As ; cos =
 0.1 2a
Fundamental frequency of string oscillations is  = 60º
 N sin60º = mg
v 20 100 2 a
n0 = = = Hz N cos60º = m
2e 0.6 3 2
Thus string will be in resonance with a turning fork
of frequency.
100 200 400
nf = Hz, Hz, 100 Hz, Hz, … .
3 3 3
Here rider will not oscillate at all only if it is at a
node of stationary wave in all other cases of
resonance and non-resonance it will vibrate at the
l
frequency of tuning fork. At a distance from one
3 w.r.t. wire
end node will appear at 3rd, 6th, 9th or similar higher
Harmonics i.e. at frequencies 100 Hz, 200 Hz, ... 2g
 tan60º =
If string is divided in odd no. of segments, these 2 a
segments can never resonate simultaneously
2g
hence at the location of rider, antinode is never 2 =
obtained at any frequency. a 3

DPPS FILE # 162

HAVE CONTROL ¾® HAVE PATIENCE ¾® HAVE CONFIDENCE Þ 100% SUCCESS


3. U(x) = x2 – 4x i.e. bc and da are constant volume process
F=0 (A) and (B) are true.
Temp. in cd process is greater than ab.
dU( x )
=0 Net work done by the gas in the cycle is
dx negative, as is clear by the PV-diagram.
2x – 4 = 0 x = 2

d2U
 = M RT
0
= 2>0
dx 2
P R
i.e. U is minimum hence x = 2 is a point of stable   M T
0
equilibrium.

7. Frequency of horn directly heard by observer


3mg 3
4. Acceleration of block AB = = g ; v  v0
3m  m 4 f
v  vc
2 mg 2g
acceleration of block CD = 2 m  m = v
3 Frequency of echo = v  v f
c
Acceleration of image in mirror AB Frequency of echo of horn as heard by observer.
= 2 acceleration of mirror
v  v  v0 
f.  
  3g  3 v  vc  v 
= 2.   = g
 4  2 Frquency of Beats :

 2g   1 1 
Acceleration of image in mirror CD = 2.   = (v + v0) f  v  v  v  v 
 3   c c 

4g 2v c ( v  v 0 )
= = f
3 ( v 2  v c2 )
 Acceleration of the two image w.r.t. each other
8. Applying work energy theorem to body
4g   3g  17g KE = work done by forces delivering power P
=   = .
3  2  6 4 4
= 2
 Pdt =  3t dt = 56 J
5. For equilibrium NA cos 60° + NB cos 30° = Mg t 2 2
and Na sin 60° = NB sin 30° Ans. 56 J
Mg
On solving NB = 3 NA ; NA = 9. (A) s, (B) p, (C) s, (D) q
2  
v A  î  a t = î  (2 î  ĵ ) (2)

 P R = 5 î  2 ĵ
6. r = M RT  
 M0
T 
0 v A   5 î  2 ĵ
Slope of the curve  Temperature   
v A , A = v A   v A  10 î
Hence cd and ab are isothermal processes.
  
1 v B  (  î  3 ĵ ) , v B  î  3 ĵ so v B , B  2 î

V For particle C (d .k C d sfy ,)
P dv y
= 2t
d dt
 vy – 6 = t 2  vy = 6 + 4 = 10
a  
c v C  5 î  10 ĵ , v C   5 î  10 ĵ

d so v C , C  10 î
V2 V1 V 
 
Equivalent PV diagram. v D  3 î  ĵ , v D  3 î  ĵ , v D , D  6 î

DPPS FILE # 163

HAVE CONTROL ¾® HAVE PATIENCE ¾® HAVE CONFIDENCE Þ 100% SUCCESS


5
DPP NO. - 4
=  10t.Vdt
0
(ds = vdt)
1. (A)
The slope of isothermal curve at point of intersection 5
t2 t2
dP P =  10 t. dt (V = adt  tdt  )
is =– = tan 135° ...(1) 0
2   2
dV V
5
The slope of adiabatic curve at point of intersection is 3

dP
=  5t dt
P 0
=– = tan 121º ....(2)
dV V 5
t4  5 625  5
from (1) and (2) 5
=  4  = 625  0 =
 =tan 59° = 1.66 = 5/3   0 4 4
 gas is monoatomic

2. Range = 10 m.
For point where particle strikes line PQ

5.

 x coordinate = 10 cos 37° + 2 = 10m


y coordinate = 10 sin 37° = 6m
z coordinate = 0m From figure if man moves from source to point A
 70 70 
   70cm  . Then he can see image
4.  2 2 
If man moves from source to point B
 70 70 
   70cm  . then he can not loose sight of
 2 2 
fmax =  × 3g image.
= 0.5 × 30 = 15 N
6. The equation of wave moving in negative x-direction,
block A starts sliding when friction force becomes
assuming origin of position at x = 2 and origin of time
max. i.e. fmax = 15
(i.e. initial time) at t = 1 sec.
at that instant (F.B. D.) y = 0.1 sin (4t + 8x)
Shifting the origin of position to left by 2m, that is, to
x = 0. Also shifting the origin of time backwards by 1
sec, that is to t = 0 sec.
y = 0.1 sin [(4t + 8(x – 2)]

both will move with same acceleration 7. As given the particle at x = 2 is at mean position at t
So 15 = 3a  a = 5m/s2 = 1 sec.
F – 15 = 7a  its velocity v = A = 4 × 0.1 = 0.4  m/s.
10t – 15 = 7 × 5
10t = 50 2 2 1
 t = 5 sec 8. Time period of oscillation T =   sec.
 4 2
Work done by friction in 5 seconds
T
Hence at t = 1.125 sec, that is, at seconds after
W=  F.ds 4
t = 1 second, the particle is at rest at extreme position.
F 10 t Hence instantaneous power at x = 2 at t = 1.125 sec
t)
=  10t.ds (a = 
m 10 is zero.

DPPS FILE # 164

HAVE CONTROL ¾® HAVE PATIENCE ¾® HAVE CONFIDENCE Þ 100% SUCCESS


DPP NO. - 5

6.
1.

Vefflux = 2gh
MR 2
M.I. about ‘O’ is
2 ( 4  h)2
time of fall t = g
2
MR
By parallel-axis theorem :
2
x = Vefflux t = 2 h( 4  h)
2
 4R  the roots of x are (0,4) and the maximum of x is at h
= cm + M  . 2
= 2.
 3 
The permitted value of h is 0 to 1 clearly h = 1 will
MR 2 4R 
2 give the

 cm = M  2.  1 maximum value of x in this interval.
2  3 
Aliter Solution:
If the column of water itself were from ground upto a
A height of 4m, h = 2m would give the maximum range
i x. Farther the hole is from this midpoint, lower the
i range. Here the nearest point possible to this midpoint
i C is the base of the container. Hence h = 1m.
2. B

3
7. W = Area under the curve = P V
2 1 1
In the figure i + i = 90°
 i = 45°

3. Slope of graph is greater in the solid state i.e.,


V 2  2 V1
temperature is rising faster, hence lower heat
P2  2 P1
capacity.
The transition from solid to liquid state takes lesser
time, hence latent heat is smaller. and P1V1 = nRT 1

3
4. W = Px w . P1V1
2
Therefore nRT =
 h h  1 P1V1
= P  
 sin 30 º sin 60 º 
8. Q = dU + W
 1  2[ 3  1] dU = nCv dT
= 2Ph 1   = Ph
 3 3 For final state P2V2 = 2P1 2V1
= 4P1V1 = nR(4T 1)
2( 3  1)( 3  1)( 3 ) Hence final temp. is 4T 1
=
3 3 9
dU = n . R . 3T 1 = nRT 1
= 4 J Ans. 2 2

3 9
5. Angular speed of reflected light = 0 rps Q= . nRT 1 + nRT 1 = 6nRT 1
2 2
There is no change in angular of incidence due to
rotation of mirror. Ans. zero Q
nRT1 = 6

DPPS FILE # 165

HAVE CONTROL ¾® HAVE PATIENCE ¾® HAVE CONFIDENCE Þ 100% SUCCESS


9. nC T = Q  nCT = 6n RT 1 3. (C) For anti-clockwise motion, speed at the highest
dT = 4T 1 – T 1 = 3T 1
n . C . 3T 1 = 6nRT 1 point should be gR . Conserving energy at (1) & (2) :
C
=2
R
10. (A) p (B) q (C) p,q (D) s
(A) If resultant force is zero,

Psystem will be constant.
(B) If resultant torque is zero,
1
 mv a2
L system will be constant. 2

(C) If external forces are absent, both Psystem and

will be constant.
L system
(D) If no non conservative force acts, total
mechanical energy of system will be constant. R 1
= mg  m(gR )
2 2
 va2 = gR + gR = 2gR
DPP NO. - 6  va

= 2gR
1. For clock-wise motion, the bob must have atleast
that much speed initially, so that the string must not
v = 0 + gt become loose any where until it reaches the peg B.
 t = 0.5 sec At the initial position :
After first collision :
Speed becomes 5 (0.5) = 2.5 m/s mv c2
t1 = 2 (0.25) = 0.5 T + mgcos600 = ;
R
t2 = 2 (0.125) = 0.25
t3 = 0.125 and so on
[where ti is the time taken to complete the ith to and
fro motion after collision]
Total time = 0.5 + [0.5 + 0.25 + 0.125 + ...]
0 .5
= 0.5 + (Since above is a G.P. with
1  0 .5
a = 0.5 and r = 0.5)
= 0.5 + 1 = 1.5 sec.

VC being the initial speed in clockwise direction.


2. For VC min : Put T = 0 ;
O
R
53º
A
R–R cos53 37º gR
=2R/5 B  VC =
R–R cos37=
R/5
2
Reference line
37º
g
g cos37
gR
By energy conservation between A & B  VC/Va = 2 = 1
2gR 2
2R MgR 1
 Mg +0= + MV2
5 5 2
 VC : Va = 1 : 2 Ans.
2gR
V=
5 6. Using newton’s formula
Now, radius of curvature r
xy = f2
V2 2gR / 5 R  20 y = (10)2
=  
ar g cos 37 2  y = 5 cm.

DPPS FILE # 166

HAVE CONTROL ¾® HAVE PATIENCE ¾® HAVE CONFIDENCE Þ 100% SUCCESS


Sol. 7 to 9 V is decreasing, W is negative.
As negative work in part-II is greater than positive
 1  1  work in part-I, net work during the process is
The shift due to slab is t 1    3  1  
    1 .5  negative.
= 1cm towards left. Hence the object appears to mirror Using PV = nRT and as Vremains same for initial
at a distance 61 – 1 = 60 cm. and final points of the process, it is obvious that
final temp. is greater than initial temperature as
1 1 1 pressure has increased. Therefore dU is positive.
From mirror formula   we get
v u f Hence option (S) is connected with (A).
v =20 cm. Similar arguments can be applied to other graphs.

f =15cm =1.5 DPP NO. - 7


1. Particle is starting from rest, i.e. from one of its
1cm I object extreme position.
y
1cm

x0 4A/5 A/5
t = 3cm P
x

A

Q
Hence the mirror forms the image at v = 20 cms
towards right. The slab again causes a shift of 1cm
A
towards right. hence the final image is formed at a As particle moves a distance , we can represent
5
distance of 21 cm from pole.
it on a circle as shown.
Shifting of slab towards left does no cause any change
4A / 5 4
to position of final image . cos  = 
The slab only causes apparent shift, but does not A 5
cause any change to velocity of image. Hence the 4
 = cos –1  
velocity of image is only due to mirror. The object 5
appears at a distance u= 60 cm from mirror and mirror
4
forms its image at v=20 cm. Hence the velocity of t = cos –1  
5
image is
2 2 1 4
 v   20  t= cos –1  
=   × velocity of object=    18  5
 u   60 
= 2 m/s towards right T 4
= cos –1  
2 5
10. (A) s (B) q (C) r (D) q Method :
in (A), V is on vertical axis. As starts from rest i.e. from extreme position x =
A sin (t +  )
At t = 0 ; x = A

Part-I  =
2
A
 A– = A cos t
5
As V is icreasing, W is positive. 4
V = cos t
5
Part-II
4
 t = cos –1
5

T 4
P t= cos –1  
2 5

DPPS FILE # 167

HAVE CONTROL ¾® HAVE PATIENCE ¾® HAVE CONFIDENCE Þ 100% SUCCESS


2. Frictional force along the in upward direction
3
= 10 g sin – 30 = 30 Nt  uy = gt
N = log cos = 80 Nt 2

1 2
Also, 10 = uyt – gt = gt2
2
 t = 1s
 uy = 15 m/s

u2 y 225 45
 h= = = m.
2g 20 4
Direction of R is along OA.
45
3. Let v1 and v2 be the velocity of efflux from square and  Maximum height attained = 20 +
4
circular hole respectively. S1 and S2 be cross-section
areas of square and circular holes. 125
= m.
4
y
2R v1
4y
6. Ans. d = 4000 mm
L v2

v1 = 8gy and v2 = 2g( y )


The volume of water coming out of square and circular
hole per second is
Q1 = v1S1 = 8gy L2 ; Q2 = v2S2 = 2gy R2
 Q1 = Q2

 R= 2 .L

4. (D) F + f = ma .... (1)
a
Also ; FR – fR = 
R
F – f = ma .... (2)
[ = mR2 ]
3
From (1) & (2) sin 60º = sinr  r = 45º
2
f = 0.
 S = h = 1m
5. tAB = t y = H tan600 = 3m
tBC = 2t  x= S + y = 4m = 4000 mm
So, for ABC part,
7. There is only one point image corresponding to a point
object, as long as the object lies on the water surface
(principal axis of the mirror). Any object lying at some
uy
u B distance from the princpal axis results in multiple
h
A
10m
C image points.
20m 20m
8. If light rays diverge outward (forming a virtual image
behind the mirror) after reflection, there is no refraction
Time of flight, at water surface after reflection. This is the case when
2u y the object lies between the focus and the pole.
tAC = 3t =
g

DPPS FILE # 168

HAVE CONTROL ¾® HAVE PATIENCE ¾® HAVE CONFIDENCE Þ 100% SUCCESS


5. From figure , the velocity of approach (Vcos)
DPP NO. - 8 decrease as the source comes closer (as 
t t increases).And the velocity of separation also
dx
 .dt  dx increases as will decrease.
dt
0 0 Hence the frequency of sound as heared by the
1. average speed v = =
t t observed decreases continuously 

x t   x 0  A cos  / 6 – 1 3 A
=
t
=
 / 6
=

3 –2 
since particle does not change it's direction in the
given interval , average speed
3 A
= v =


2– 3 
2. Since the block slides down the incline with uniform
velocity, net force on it must be zero. Hence mg sin ROC =150cm
must balance the frictional force ‘f’ on the block.
S1 S2 S3
Therefore f = mg sin = 5 10 ½ = 25 N.
=4/3 =3/2 =1 =3/2 =4/3
3. (C) Angular momentum will be conserved if the net O
torque is zero . 6. 20cm
Now for the sphere to move down: 10cm
45cm 24cm 54cm
mg sin >  mg cos

Apparent shift in the object O dut to three slabes S1,S2

4
and S3 with respect to the medium of  = is given by
Let x be the perpendicular distance of the point 3
(as shown in figure) about which torque remains :
zero. Shift
for  = 0 ; x > R as shown
     
 1   1   1 
451    241    541  
=  3/2   1   3/2 
     
 4/3   4/3   4/3 

 8  4  8
Note: As mgsin > mgcos, the point should be Shift = 45  1 -   24 1 -   54  1 -  .
 9  3  9
inside the sphere.
Shift = 5 + (–8) + 6 = 3 cm
4.  = 2 = 3m  Unet = 150 cm and ROC = 150 cm.
Equation of standing wave Hence image will be formed on the object itself.
y = 2A sin kx cos t
y = A as amplitude is 2A. 7. The moment of inertia of all seven rods parallel to
A = 2A sin kx AB and not lying on AB is the moment of inertia of
2  all five rods lying on AB = 0
x=
 6 The moment of inertia of all 18 rods perpendicular
1
 x1 = m 2
4 to AB is = 18 () = 6 3
3
2 5
and .x= Hence net MI of rod about
 6
AB = 7 3 + 6 3 = 13  3 Ans.
 x 2 = 1.25 m  x 2 – x 1 = 1m

DPPS FILE # 169

HAVE CONTROL ¾® HAVE PATIENCE ¾® HAVE CONFIDENCE Þ 100% SUCCESS


3. For first resonance with 400 Hz tuning fork
 2 1   2  1 
8.    V V
v u  R  eq = 4 f = = (19 + 1) = 20 cm
0 4( 400 )
(2 – 1) is +ve and R is – ve if u is –ve, v will always
If we use 1600 Hz tuning fork
be –ve
i.e. for real object image is always virtual. V V 20
  = 5 cm
4f0 4  (1600 ) 4
Sol. 9. to 10. for Resonance
Consider object on left side of spherical surface
V 3V 5V 7V
seperating two media. eq = , , , , ....
4 f0 4 f0 4 f0 4 f0
If real object is in rarer media i.e., n1 < n2 400 Hz

n2 n 2  n1 n
Then   1 = – ve
v ( u) ( R )
20 cm

-----
-----
-----
-----
-----
-----
Hence image shall be virtual for a real object -----
-----
lying
on concave side with rarer media ....(1) 1 cm +  = 5 cm , 15 cm , 25 cm , 35 cm ,
If real object is in denser media i.e., n1 > n2 45 cm .....
n2 (n1  n 2 ) n n  n 2 n1  = 4 cm , 14 cm , 24 cm , 34 cm , 44 cm .....
  1 = 1 
v ( u) ( R ) u R water level should be further lowered by
24 – 19 = 5 cm  34 – 19 = 15 cm
n1  n 2 n1
 Image is real if > or u
u R 4 As rays are parallel to the principal axis, image is
(n1  n 2 ) R created by lens at the focus.
< .... (2) By placing of glass-slab,
n1
 1
 n1  n 2  Shift =  1   .t
and image is virtual if u >  n  R .... (3)  µ
 1 
 1 
From statements 1, 2 and 3 we can easily = 1   3 = 1 cm.
conclude the answers.  1 .5 
Irrespective of separation,
Image is shifted to the right by 1 cm.
DPP NO. - 9 Total distance from lens 10 + 1 = 11 cm
Ans.
1. As  = 0, angular momentum remains conseved
:
5.
 300R 2  300R 2 
 L = 0 
   =   30R 2  .
 2  0

 2



n 2 n1 n 2  n1
 150 0 = 180    = 5/6 0 Ans.  =
v u R
1 1.5 1  1.5
 =
v  2R R
2.  v = –4R

n1v 1.5  ( 4R)


Let v = velo. of rain m= =
n 2u 1 (2R)
Possible values of  are
–30º <  < 90º . m = 3.

DPPS FILE # 170

HAVE CONTROL ¾® HAVE PATIENCE ¾® HAVE CONFIDENCE Þ 100% SUCCESS


6. If F = 20 N, 10 kg block will not move and it would 3. Time of flight of projectile depends on vertical
not press 5 kg block So N = 0. component of velocity and not on the horizontal
component. Collision of the stone with the vertical
wall changes only the horizontal component of
velocity of stone.
Thus the total time of flight in absence of wall is
also T = 1 + 3 = 4sec
2u y
 =4
g
or uy = 20 m/s
8. If F = 50 N, force on 5 kg block = 10 N
u 2y 400
or Hmax = = = 20 metres.
2g 20

So friction force = 10 N 4.
min > C
9. Until the 10 kg block is sticked with ground sin min > sin C
(... F = 40 N), Rd 1
No force will be flet by 5 kg block. After F = 40 N, >
R n
the friction
 R n – dn > R
force on 5 kg increases, till F = 60 N, and after
that, the kinetic friction start acting on 5 kg block, nd
 R>
n 1

2 . 4mm
R>
2 1
R > 8 mm

which will be constant (20N) i 60° i


5.
DPP NO. - 10 r c c r
60° 60°
 v   v  B C
1. (C)    v  v  f0 = 2 Hz
 
 v  v s   s 
Total Deviation = (i – r) + (180 – 2c) + (i – r) = 112°
vs = 0.5 m/s as r = 60 – c
2i – 120 + 2c + 180 – 2c = 112°
2. Let velocity of c.m. of sphere be v. The velocity of
 2i = 52°, i = 26°
the plank = 2v.
1 7. F = a sin t
Kinetic energy of plank = × m × (2v)2 = 2mv2
2 dv dv a
m = a sin t  = sin tsin t
1 dt dt m
Kinetic energy of cylinder = mv2
2 v t
dv 
1  1 mR 2  2    sin t.dt
+ +  0 0
2 2 
a a a
1 2 1 3 1 v=  cos t = [1 – cos t]
= mv 1   = . mv 2 m m m
2  2  2 2
since cos t  1
K.E. of plank 2mv 2 8 hence direction of velocity will not change.
 = 3 = . Ans. Will not change
K.E. of sphere mv 2 3
4

DPPS FILE # 171

HAVE CONTROL ¾® HAVE PATIENCE ¾® HAVE CONFIDENCE Þ 100% SUCCESS


8. From formula for refraction at curved surface 6.

n 2 n1 n 2  n1
  v = – 4 cm
v u R
 image is formed in denser medium at a distance
4 cm from pole.

v n1
9. Size of image =  × size of object
u n2

4 3 6
=  2 = cm. Friction force acting on car will be resultant of the
20 1 5
components shown in the diagram.
fg and fC are components of friction force that balance
gravitational pull and provides centripetal force
respectively
i r The resultant of fg and fc can be horizontal only for a
point at BC and CD
10. C and not for AB and BD
n=3 n=1
7. As work done in state () is more than in state ()
40 cm from pole in the medium of refractive index 1,
virtual, erect and 4 cm in size.
A ray incident from object O is in denser medium and
is refracted into rarer medium.
 r > i Hence always virtual image is formed.

DPP NO. - 11

1. When object moves normal to the mirror, image 8. Image due to plane mirror will form at a distance of
velocity will be opposite to it. When object moves 20 cm left of the mirror.
Since image formed by two mirrrors lie adjacent to
parallel to the mirror, image velocity will be in the
each other.
same direction.
For convex mirror, image position is 15 cm towards
left.
2. As the object moves from infinity to centre of u = – 25 cm
curvature, the distance between object and image v = + 15 cm
reduces from infinity to zero. 1
1 1 2
As the object moves from centre of curvature to using + = =
v u f R
focus, the distance between object and image
increases from zero to infinity. 1 1 2
– =
As the object moves from focus to pole, the 15 25 R
distance between object and its image reduces R = 75 cm.
from infinity to zero. Hence the distance between Ans. R = 75 cm.
object and its image shall be 40 cm three times.
9-10. Suppose velocity of ring
3. Force on bottom surface = gH × A

4. At any time

v = [(u cos) î + (u sin – gt) ĵ ]





P = F.v  ( mg ĵ ). u cos  î  (u sin   gt ) ĵ  When it starts pure rolling is v. Angular momentum
can be constant about point of contact as there is
= mg2 t – mgu sin
no external torque acting about it. Thus
Hence, power varies linearly with time.

DPPS FILE # 172

HAVE CONTROL ¾® HAVE PATIENCE ¾® HAVE CONFIDENCE Þ 100% SUCCESS


4. Resultant Displacement
v y = y1 + y2
2
m.2.R = m.R   + mvR
R for y to be zero
 v = 1m/s y=0
using v = u + at (2x – 3t)2 = (2x + 3t – 6)2
1 = 2 + (–4) t 3
 t = 1/4 sec. [ Ans.: 1 m/s, 1/4 sec. ] on solving (x – ) (t – 1) = 0
2
DPP NO. - 12 Therefore,
3
at x = , resultant displacement is zero for all
2
values of t.

i' y
1. 5. v
i

u=50 30°

 = 1 + 2 = (180 – 2i) + (180 – 2i' h


= 360 – 2 (i + i') 45°
o x
 = 360º – 2
 = 360º – 2(90º) = 180º . h = height of the point where velocity makes 30º with
horizontal.
2. The only possibility is by reflection from concave As the horizontal component of velocity remain same
mirror as shown. 50 cos45° = v cos30°
2
v = 50
3
Now by equation
v2 = u2 + 2ay y
2
 50  2  = 502 – 2gxh
 

 3 

2
 dv   2gh = 502 – 502 ×
3. (A) Area under P–x graph =  p dx =  m  v dx
 dt 
3
1
 2gh = × 502
v v 3
2  mv 3  10 2500 125
= mv dV =  3  =
 (v3 – 1)  h= =
1  1 73 60 3
125
1 h= m above point of projection
from graph ; area = (2 + 4) × 10 = 30 3
2
6. As Rod is in linear motion only (there’s no rotation
10 of the rod), Net torque about COM must be zero.
 (v3 – 1) = 30
73 Hence
 v = 4 m/s
 
– F2   b  = 0

ALITER : F 1.
2
........(1)
2  
from graph
P = 0.2 x + 2 also for linear motion.
F2 – F1 = ma ........(2)
dv solving (1) and (2)
or mv v = 0.2 x + 2
dx
or mv2 dv = (0.2 x + 2) dx
Now integrate both sides,
v 10

 mv 2 dv =  (0.2 x  2) dx
1 1

 v = 4 m/s.
2F2b
= Ans.
ma

DPPS FILE # 173

HAVE CONTROL ¾® HAVE PATIENCE ¾® HAVE CONFIDENCE Þ 100% SUCCESS


7. for M1 :  u = – 30, f = – 20
1 1 1
 + =  v = – 60 cm
v  30  20

v 60
m1 = – =– = – 2.
u  30

where U is the change in the internal energy of


the gas; and W is work, done by the gas. For one
mole of the monatomic ideal gas U = 3/2R T.
Work equals the area under the graph P vs. V
Therefore, for the process from the initial state with
P1V1 = 3/2 RT 1 to the state with P,V,T the heat
given to system is
Q = (3/2) R (T – T 1) + (1/2) (P + P1 (V – V1)
The image By M1 is PQ
for M2 : u = + 10 , f = + 20 3 1
= (PV – P1V1) + (PV + P1V + PV1
2 2
1 1 1 – P1V1) .... (3)
 + =  v = – 20 cm
v 10 20
1 1
= 2PV + P 1V – PV1 – 2P1V1
20 2 2
m2 = – =+2 from equation 1 and 3 we get
10
now the size of final image is = 8 cm. Ans. P0 2 5  5 V1 
Q = 2 V V  2 P0 V  2P0 V1  4  V 
0  0 

The process switches from endothermic to exothermic


dQ
as changes from positive to negative, that is
dV
dQ 5
at = 0. Solving we get V = V0
dV 8
distance of P from AB = 1 cm. Ans.
distance of Q from AB = 7 cm. Ans.
8, 9, 10. From The P–V graph, the relation between P DPP NO. - 13
Po
and V is P   V V  P0 .... (1) 1.
o

Also the ideal gas state equation for one mole is In 20 seconds , distance travelled by the Source
PV =RT .... (2) is :
Po  V  1
From equation (1) and (2) is T  V 1   S= (1.1) (20)2 = 220 m.
R  Vo  2
Let, t = 0 be the starting time.
Hence the graph of T vs. V is a parabola given by The sound wave started at t = 0 from 'A' reaches
 330 
the observer at 'B' after   = 1sec. ie. observer
 330 
started hearing the sound at t = 1 sec. At t = 20
sec. the source reaches at 'C' 220 m from 'A'.
The last sound wave starting from 'C' reaches 'B'
 110  1
afrer   = sec < 1 sec.
Vo  330  3
Obviously T is m axim um at V= . There
2 Hence , the observer do not hear the sound for
whole 20 sec.
P0 V0 but for:
maximum value of T is 4 R
 1 58
T =  19   sec = sec.
Q = U + W  3 3

DPPS FILE # 174

HAVE CONTROL ¾® HAVE PATIENCE ¾® HAVE CONFIDENCE Þ 100% SUCCESS


2. The retardation is given by 6. T.K.E. of disc [M.Bank_Rotation_7.120]
dv
= – av2 1 1
dt = mv 2  2
2 2
integrating between proper limits
t 2
v
dv 1 1 mr 2  v 
= mv 2    
 –
 v2 =  a dt 2 2 2 r
u 0

1 1 3
or = at + = mv 2
v u 4
Velocity of particles of upper half is more than that of
dt 1
 = at + lower half hence kinetic energy of upper half will be
dx u

u dt 3
 dx = more than mv2 .
1  aut 8
integrating between proper limits
K
s t
7.  = = 10 rad/s
u dt m
  dx =  1  aut
0 0 2 2
T=  s
 10
1
 S= n (1 + aut)
a

3. In the frame of ring (inertial w.r.t. earth), the initial Maximum speed will be at the natural length of
velocity of the bead is v at the lowest position.
2 
the spring at T/4 = = s.
10  4 20

T 
Time taken to cover 0.1 m is  s
4 20

1
Time taken to cover × 0.1m is
The condition for bead to complete the vertical circle 2
is, its speed at top position
vtop  0 T 2 2 2 
 =   s
From conservation of energy 4 3 10  4 3 30
1 1
m v 2top + mg (2R) = mv2
2 2 8. Compression is maximum when both blocks move
or v = 4 gR with same velocity V.
By cons. of momentum
5. Since angular velocity is constant, acceleration of
centre of m ass of disc is zero. Hence the m1v 1  m 2 v 2
V= m1  m 2 = 5 m/s
magnitude of acceleration of point S is 2x where
 is angular speed of disc and x is the distance of
The change in K.E. = k f – k i = – 35 J
S from centre. Therefore the graph is
This is stored as spring PE

1 2 2 K
Therefore kx = K  x =
2 k

on solving x = 0.25 m = 25 cm.

DPPS FILE # 175

HAVE CONTROL ¾® HAVE PATIENCE ¾® HAVE CONFIDENCE Þ 100% SUCCESS


9 to 11 3. P.E. is maximum at extreme position and minimum
Magnification is negative, therefore lens is at mean position.
Time to go from extreme position to mean position
convex. v = m.u
u is negative and v is positive u T
is, t = ; where T is time period of SHM
4
1 1 1
 = f  (i) T
20 10 1 5s=
4
1 1 1  T = 20 s.
 = f  (ii)
30 10 2   
4. d  d f  di = 8 î  6 ĵ
1 1 1 1 1  
 = f f =  (iii) W = F . d = 24 + 24 = 48 J
v 10 1 2 f
W
For combination Pav = = 8 Watt
t
v 60 / 11 6
m= = = 5. At max. extension both should move with equal
u  10 11
velocity.

k = 1120 N/m
DPP NO. - 14
5kg 2kg
1. using angular momentum conservation
Li = 0  By momentum conservation,
(5 × 3) + (2 × 10) = (5 + 2)V
Lt = mvR – 
V = 5 m/sec.
mvR =  Now, by energy conservation

 1 1 1 1 1 2
=   (v + R)t = 2R 5 × 32 + × 2 × 102 = (5 + 2)V2 + kx
2 2 2 2 2
Put V and k
 1  1
 1   2   2  2 t = 2 sec.  x max = m = 25 cm.
 2  4
Also first maximum compression occurs at ;
2. min = i + e–A 3T 3 
t= = 2
min = A then 4 4 k
2A = i + e in case of min i = e
3 10 3
A = 2 = sec.
2A = 2i r1 = r2 = 4 7  1120 56
2
m1m 2
i = A = 90° (where   reduced mass,  = m  m )
then 1 sin i = n sin r1 1 2

A
sin A = n sin
2
6.
A A A
2 sin cos = n sin For equilibrium of all the changes net force on
2 2 2 each charge should be zero.
A 4KqQ
2 cos =n  KqQ – = 0 (Net force on Q)
2 d 2 ( x  d) 2
when A = 90° = imax  d = x/3
4Kq 2
& KqQ + = 0 (Net force on A )
then nmin = 2 d2 x2
i = A = 0 nmax = 2 4
 Q =  q & d = x
9 3

DPPS FILE # 176

HAVE CONTROL ¾® HAVE PATIENCE ¾® HAVE CONFIDENCE Þ 100% SUCCESS


9. Distance of image of eye from fish in upward
direction

7. d 60 4
d= n + 30 = = 60 × + 30
rel   3
 1 
 4 
 3
v1 = 2 g (h  H / 2)
= 110 cm
v2 = 2g h
 By continuity equation 10. Distance of first image of eye from the refraction
at water surface
 dh 
A    = a (v + v )
 dt 
1 2
60
v= = 80
 
 1 
 dh   4 
 A    = a
 dt 
 2 g (h  H / 2)  2 g h   3

H/2 Distance of image of eye from mirror due to


t
A dh refraction = 80 + 60 = 140
or   =  dt
a 2g H h h  H/2 0 Distance of second image in downward direction
from mirror = 140 cm
2A H Distance of second image from fish in downward
 t=
3a
 2 1  g
Ans.
direction = 140 + 30 = 170 cm

8. Equation of standing wave can be written as DPP NO. - 15


Y = 2 sin (kx + ) cos t
because the particles at t = 0 are at extreme position. 1. R = 4 2  4 2  2.44 cos 120  = 4N
2 2 
k= = =
 4 2

1
From the graph it is clear that x = ,
2
Amplitude = 0

 1  
 2 sin  .   = 0     0 or 
2 2  4 2. The force exerted by film on wire or thread depends
only on the nature of material of the film and not on

we will select     to suit the initial condition its surface area. Hence the radius of circle formed by
4
elastic thread does not change.
3
 =
4
3. N = Mg(1 – cos)
 3  5
 y = 2 sin  x   cos t
2 4  2

 3 5 
= sin  x   t
2 4 2 

 3 5 
+ sin  x   t
 2 4 2 
Mg sin > Mg (1 – cos)

DPPS FILE # 177

HAVE CONTROL ¾® HAVE PATIENCE ¾® HAVE CONFIDENCE Þ 100% SUCCESS


4. At time t1, velocity of the particle is negative i.e. f = MC (a – b) ....(3)
going towards –X m. From the graph, at time t1, its for plank
speed is decreasing. Therefore particle lies in 20 – f = 4.a ......(4)
between –Xm and 0. putting MC = 2kg
At time t2, velocity is positive and its magnitude is f=b 2f = 2 (a – b)
less than maximum i.e. it has yet not crossed O.
It lies in between –X m and 0. 2a
 f=
Phase of particle at time t1 is (180 + 1). 3
Phase of particle at time t2 is (270 + 2)
Phase difference is 90 + (2 – 1) 2a
using in (4) 20 – = 4a
2 – 1 can be negative making  < 90° but can 3
not be more than 90°.
30
 a= m/s 2
7
A
5. × 100 20
A and b = m/s 2
7
  
= 2  × 100 10
A  Acceleration of cylinder = a – b = m/s 2
7
 % increase in Area
= 2 × 0.2 = 0.4 10 30
[ Ans.: a1= m/s2, a2 = m/s2 ]
V
7 7
× 100 = 3 × 0.2 = 0.6 % 8. Apperent distance
V
Since  =   T d1 d2
(t) = n / n + n / n + d3
1 3 2 3

× 100
 but n1 = n3 = 1 & n2 = 3/2
= T × 100
d1 d2
= 0.2 d= + + d3
1 3/2
  = 0.25 × 10–4 / ºC
but d2 and d3 are constant when only object is
moving.
area under F – T graph
6. Change in velocity = So d = d1 +0 + 0  vI = v0 = 12 cm/sec.
mass
d1 d2
40  (–10 ) 9. d = n / n + n / n + d3
= = 6 m/s 1 3 2 3
5

1 12 9 4
(5) 62 = 90 J
W F = K.E. = = + 3/2 4/3 +
2 1/ 4 / 3 / 1
7. Suppose acceleration of cylinder wrt plank is ‘b’
wrt plank. 4 8
= 12 × + +4
3 1

= 16 + 8 + 4
= 28 cm.

d1 d2
10. d = + + d3
1/ 4 / 3 9/8
As there’s no slipping b = R ....(1)
Equation of Rotation motion f. R = . 4v 1
v = +0+0
3
MCR 2 b MCb
 f= . = ....(2)
2. R R 2 4
v = × 12 = 16 cm/sec.
Linear motion of cylinder 3

DPPS FILE # 178

HAVE CONTROL ¾® HAVE PATIENCE ¾® HAVE CONFIDENCE Þ 100% SUCCESS


DPP NO. - 16 RT
5. Velocity of sound in air (V) =
M
2. (C)
 V2T (in kelvin)
not V2  T (in 0C)
Oil Hence (B) is incorrect.
Velocity of transverse wave in a string :
Mercury
T
V= = V2  T

Weight = Buoyant force Hence (C) is a correct graph.
V V
Vmg =  g+  6. If we displace the electron slightly toward
2 Hg 2 oil
x direction, it will thrown away toward right.
 Hg   oil 13.6  0.8 So eql. is unstable along x direction.
rm = =
2 2 If we displace the electron slightly towards
14.4 y direction, No extra force will act. So eql. is
= = 7.2 neutral along y axis
2
If we displace the electron toward z direction, it
3. The friction force on coin just before coin is to slip will will be attracted and try to come to eql. positron.
be : So eql. is stable along z direction.
f = µs mg
f = µs mg 7. [ Ans. 1:1 ]
Normal reaction on the coin ; N = mg
The resultant reaction by disk to the coin is 8. As it is posible only when, when light reflects from P1
after refraction from the plate P2 of light coming from
light source.

= N2  f 2

= (mg)2   2s (mg)2
First image will form due to reflection from the right
= mg 1  2 surface of P1 . As light ray is falling on P1 from 11 cm
so it will form image at 11 cm in left. So, distance of
9 first image from insect is 11 + 3 = 14 cm.
= 40 × 10–3 × 10 × 1 = 0.5 N
16
9. Second image will form due to reflection on left
4. Applying work–energy theorem, surface of P1.
W all forces = K.E.
W g + W N + W f = K.E.
 ( 1 × 10 ×1 ) + 0 + W f
1
= (1)[(2)2 – (0)2]
2
10 + W f = 2
W f = – 8J.
For left surface of P1 light seems to coming from the

DPPS FILE # 179

HAVE CONTROL ¾® HAVE PATIENCE ¾® HAVE CONFIDENCE Þ 100% SUCCESS


3. Area under the curve is equal to number of
33 39
distance = +3= cm molecules of the gas sample. Hence
2 2
1
N = . a . V0  aV0 = 2N
45 2
So, light seems to coming from cm from right
2 
1
surfaces of P1 . Vavg =
N
 v N(V ) dV
0
So, final position of second image will be
V0
1  a  2 Vavg 2
45 =
N
 C .  v
0 0
.V dV =

V
3 0

V0
=
3
2
= 3 = 15 cm.
2 
1 2
v r2ms =
N
V N( V ) dV
So, distance of second image from insect = 18 cm. 0

v0
10. Due to multiple reflections infinite image will be formed 1  a  2 Vrms 1
= V 2  .V  dV = V0
N
  V0  2
 V0 = 2
0

DPP NO. - 17 3
Area under the curve from 0.5 V0 to V0 is of total
4
area.
1.
4. VL = VV
 YLVL = YVVV

YL VV
or Y = V
V L

but VV > VL
 YL > YV
Let the radius of cylinder be R
For the cylinder to remain static, net torque
5. Y1 = A sin t
on cylinder about point P (point of contact with
Y2 = A sin(t + )  = 2/3
inclined surface) should be zero.
 Mg (OS) = mg (SQ) As, Y2 + Y1 = A
or Mg R sinq = mg R (1 – sinq) Now, solve and use the condition of maxima.
Alternate : by symmetry both should be on opposite
M sin 
or m = side of mean position at equal distance from mean
1  sin 
(for max. seperation)

6
2. (B) y(x, t = 0) = then y(x,t)
x2
6 As projection of SHM on circular path
= 2
( x  2t )

y 24
 =
t ( x  2t )3
at x = 2, t = 2

24
Vy = = – 3 m/s.   
( 2 ) 3 The phase difference from figure is  
6 6 3

DPPS FILE # 180

HAVE CONTROL ¾® HAVE PATIENCE ¾® HAVE CONFIDENCE Þ 100% SUCCESS


6. Ny Ny

Ny Ny
mg

4Ny + mg = ma
2 Put Ny from (3)
cos  
3 4m'g – 6m'a + mg = ma ..... (4)
As m' = 2kg & m = 5 kg
 8g – 12a + 5g = 5a
13g
Kq2 2 a= 
 Fnet = 3Fcos  = 3 17
a2 3 8. From equation (4) (neglecting m') a = g 

6 q2 2g
= 9. From equation (4) (neglecting m) a = 
4 0 a 2 3

6 q2 10. (A)  p, s ; (B)  r ; (C)  r ; (D)  p, s


[ Ans. ]
4 0 a 2
DPP NO. - 18
1. Los s in heat f r om calor im eter + water as
Sol. 7 to 9 temperture changes from 10°C to 0°C
7. F.B.D. of disc = m 1C110 + m 2C210
 In horizontal direction N = Nx .....(1) = 1 × 1 × 10 + 1 × 0.1 × 10 = 11 kcal
In vertical direction m'g – f – Ny = m'a.....(2) Gain in heat of ice as its temperature changes
Torque due to friction ; from –11°C to 0°C
= m 3C3 × 11 = 2 × 0.5 × 11 = 11 kcal
m' R 2 Hence ice and water will coexist at 0°C without
fR = 
2 any phase change.

Ng
f

Nx
2.

m'g

As a = R

m' R 2 a
 fR =
2 R
m' a
 f= put in (2)
2 Fe Fe / k
tan = Vg = V(  )g
3m' a
 m'g – Ny =
2
 2 .4
3m' a  k= = = 1.5
 2 .4  0 . 8
Ny = m'g – .....(3)
2
Now, F.B.D. of bar ;  2 .4
k= = = 1.5
 2 .4  0 . 8

DPPS FILE # 181

HAVE CONTROL ¾® HAVE PATIENCE ¾® HAVE CONFIDENCE Þ 100% SUCCESS


3. Because the acceleration of wedge is zero, the normal
8m 80m
reaction exerted by wedge on block is T1 = g = ................(i)
3 3
N = mg cos37° .
In resonance,
The acceleration of the block is g sin 37° along the
fwire = ftube
incline and initial velocity of the block is v = 10 m/s
horizontally towards right as shown in figure. (1)V1 (1)V2
2 1 = 4 2

 T1 
 
   ( 400)
 
= x
2( x ) 4 
2
 T1 = (16 × 104)
From (i),
The component of velocity of the block normal to the
incline is v sin 37°. Hence the displacement of the 80
m = 10–4 (16 × 104)
block normal to the incline in t = 2 second is 3
m = 0.6 kg.
3
S = v sin 37° × 2 = 10 × × 2 = 12 m.
5
 The work done by normal reaction

4 7.
W = mg cos 37° S = 100 × × 12 = 960 J
5
4. Since, the medium has not changed, speed of wave
remains same. µR = 1.52
 v = f = constant µv = 1.6
Minimum deviation condition for red is r = 30°
f 1  1 = f 2 2
 (1) sin i = (1.52) sin30°
 f11 = (1.25f1) 2
i = 50º,
( frequency increased by 25%) R = (50º) 2 – 60°
= 40°
1
 2 =
1.25
8. For violet light
  2 decreases. (1) sin 50° = (1.6) sin r
 % change in wavelength  r = 28.4°
r = 31.6° (  r + r = A )
1 (1) sin e = (1.6) sin 31.6°
1 
1   2 1.25  e = 56°,
= × 100 = × 100
1 1  v = i + e – A = 50º + 56° – 60°
= 46°
0.25 100  angular width = v – R = 6°
= × 100 = = 20%
1.25 5
9. The length of the spectrum if it is focussed on a screen
by a lens of focal length 100 cm is :
10 10
(A*) cm (B) m
3 3
6.
5 5
(C) cm (D) m
3 3

 10
 2m(2m)  Sol. if  = 100 × 6 × cm = cm
T1 = 2T0 = 2  g 180 3
 m  2m 
10. (A)  (p, r, s), (B)  (p, s),(C)  (r, s), (D)  (q, s)

DPPS FILE # 182

HAVE CONTROL ¾® HAVE PATIENCE ¾® HAVE CONFIDENCE Þ 100% SUCCESS


9. (i) By conservation of volume
DPP NO. - 19 4 × h = 4 × 2 + 2 × 1 = 10
h = 2.5m
1. The FBD of rod is Pressure at top of the object
Taking moment of force about centre of mass = P0 + 0.5 × 1000 × 10
= 1.05 × 105 N/m2
F = P1A
= 1.05 × 105 × 2 = 2.1 × 105 N

T
P1A
L L
N2 × – N1 sin × =0 .5m
4 2
N1 1 A=2m2 1m
h
 N  2 sin 
2
mg
P2A

b
4m2
2K dx b By F.B.D.
2. E =  dE = x = 2 K n
xa a T + P2A = mg = P1A
T = mg + (P1 – P2) A
b
K  x  dx = mg – (P2 – P1) A
V=  dV =  x = K (b – a).
x a
= 2 × 2000 × 10 – (.2 × 105)
= .4 × 105 – 0.2 × 105 = 0.2 × 105 N
2
E 2knb / a  lnb / a  Fb = V.wg
 = =
V kb  a  b  a  = 2 × 1000 × 10 = 0.2 × 105 N
It is also equal to net contact force by
V the liquid = P2A – P1A = 0.2 × 105N
Nx Note : Net contact force and buoyant force are
3. N same.
Ny
A B
10. (A) q,r (B) p,s (C) p,s (D) p,s
The horizontal component of velocity of Q will increase
(A) Centre of mass lies in second quadrant.
and become maximum at the top ; and will again
(B), (C) and (D) Centre of mass lies on y-axis
become same at B. Because of its greater horizontal
and below x-axis.
velocity the particle Q will reach B earlier than P
 t1 > t2 .
DPP NO. - 20
4. Given : f0 = 200 Hz. , A = 50 cm = 0.5 m, V = 340 m/s.
According to question :
1. (Moderate) Draw an incident ray along the top side
of rectangular strip,which happens to be parallel
to the principal axis. After reflection this ray passes
2 f0 Vs max through focus. Hence image of all points (for e.g.
= 20 20 O1, O2, O3, .......) on top side of the strip lie on
V
this reflected ray (at I1, I2, I3, .......) in between
2  200  0.5   focus and centre of curvature. Thus the image of
=
340
sin ce; VS max  A this strip is a triangle as shown in figure
= 34 rad/s.
O1 O2 O3

5. B,C,D direction of friction do not dpend on direc- F


tion of force but it depends on direction of relative I C
motion (velocity) I3 I
2
I1

6. a

DPPS FILE # 183

HAVE CONTROL ¾® HAVE PATIENCE ¾® HAVE CONFIDENCE Þ 100% SUCCESS


V12  V22  ..........VN2 2
2. Vrms = maximum accelesation = w2A =  3 mm
N 4
= 0.75 2 mm/sec2.
2 2 2
1  2  ..........  N 2 
= (c) k =  m–1
N  3

2 
N(N  1) (2N  1)  w=  rad/sec
= T 2
6N
  
y = 3 sin  3 x – 2 t  o 
(N  1) ( 2N  1)  
 Vrms =
6 y(x = 2, t = 0 ) = 0

V1  V2  ........  VN 1  2  ........  N  2 
Vavg = =  sin  3   0  = 0
N N  

N (N  1) N1 2 
= 2N = 2  0   or
3 3

Vrms ( 2 N  1) y
2 and ( t = 0 , x = 2) > 0
Vavg = 6 (N  1) t
4. The magnitude of phase difference between the –3   t 
points separated by distance 10 metres  cos  3 x – 2  o  > 0
2  
= k × 10 = [10 × 0.01] × 10 = 
(For x = 2, t = 0)

5. At E,  2  
 cos  3  o  < 0  o 
EE = EA – EB   3

2 /   10 9 1/   10 9  x t 
= – y = (x,t) = 3 sin  3 – 2  3 
2. 0 2. 0  

= 18, towards right. y  x t  


  cos  –  
x  3 2 3

y
 at x = 2 and t = 4 sec ; × 10–3
x

3QR 2
At D 7. Ans.
8  0 x 4
EO = EA + EB

2 /   10 9 1/   10 9 8. Let ‘u’ be the required minimum velocity. By mo-


= + = 54,
2 0 2 0 mentum conservation :
towards right. mu = (m + m)v  v = u/2.

6. (a) from y – x graph


wavelength =  = 6m
from y – t graph
Time period = T = 4 sec
1
 frequency = f = = 0.25 Hz
4
Energy equation :
wave speed = f = 0.25 × 6 = 1.50 m/s
1 1
(b) maximum velocity = 3mm ×

rad/sec mu2 = (2m)v2 + mgH.
2 2 2
= 1.5  mm/sec Substituting v = u/2 : u = 2 gH

DPPS FILE # 184

HAVE CONTROL ¾® HAVE PATIENCE ¾® HAVE CONFIDENCE Þ 100% SUCCESS


9. When the block comes to rest, the wedge continues 5. As N sin  = mg
N cos  = m2 r
u
to move at V = = gH on the smooth surface. g
2
tan  =  T 2  tan 
(since, momentum of wedge-block system remains  2r
Nsin
conserved). N


10. By work-energy theorem of the block : Ncos
– (mg) (BD) – mg(H–h) = 0 a h
Hh
BD =
  when  increases T also increases
Also T 2  r tan 
DPP NO. - 21 but r = h tan 
1. Let h = height to of water column  T 2  h tan2 
then wgh + Hg g(10–h) = Cu g10 for constant 
 h + 13.6 (10 – h) = 73 T2  h
Thus when h increases T also increases
63 = 12.6 h  h = 5 cm

9.  Electric field near point b is – 


2. Higher is the temperature greater is the most prob-
 ‘b’ should be negative electric field at x 1 is O
able velocity. which possible only if ‘a’ and ‘b’ are of opposite
sign.
3. Just before the particle transfers to inclined sur-  ‘a’ is positive
face, we resolve its velocity along and normal to the Charge b is negative and charge a is positive
plane.

ucos ucos
E at A = 0

After impact | Qa | | Qb |
Before impact
 (  x )2  ( x )2
usin 1 1

For the trajectory of the particle to sharply change 2 2


from the horizontal line to the inclined line, the impact  Qa    x1    
     1  
of the particle with inclined plane should reduce the Qb x x1 
 1  
usincomponent of velocity to zero. Hence the particle
moves up the incline with speed u cos. E at a general X
Hence as  increases, the height to which the particle
rises shall decrease. K | Qa | K | Qb |

(  x 2 ) (x2 )
4. [Moderate] = i + e – A (for minimum derivation
i = e)  1 Q 1 
 minimum deviation = 2i – A = K | Qa |   b 
2 Q a x 2 
 (   x )
60 = 2 × 60 – A   A = 60°
dE
 A  m   60  60  If E is a maximum , 0
sin  sin   dx
 2   2 
n= A
  =  60  = 3 2
sin  sin   2  x  2
2  2   3
  1  3  0
(  x )    x1  x
1 = i1 + e – A
2 2
65° = i1 + 70° – 60° or i1 = 55°    x1     x1  3
3
the  versus i curve is not parabolic (  x )3 = x   ;   x = x  
 x1   x1 

DPPS FILE # 185

HAVE CONTROL ¾® HAVE PATIENCE ¾® HAVE CONFIDENCE Þ 100% SUCCESS


2. V = V1 + V2 + V3

 x2 = 2 1 1   2Q  1  3Q 
Q
   x1  3 = 4  . + 4   +  
   1 0 R 0  R  4 0  R 
 x1 
1  2Q 
= .  
2 4 0  R 
 Qa   
Ans: x2 = ,  1   , 3. Let the body is acted upon by a force at an angle
2 Qb x
 1
   x1  3  with horizontal.
   1
 x1  FBD :

Charge b is negative and charge a is positive

10. (A) p (B) q,s (C) p (D) q,s


(A) Speed of point P changes with time
(B) Acceleration of point P is equal to
2x ( = angular speed of disc and x = OP). The
acceleration is directed from P towards O.
(C) The angle between acceleration of P F cos  =  (mg – F sin)
(constant in magnitude) and velocity of P changes
 mg
with time. Therefore, tangential acceleration of P F = cos    sin  . For min. force 
changes with time.
(D) The acceleration of lowest point is directed to- (cos +  sin) should be max.
wards centre of disc and remains constant with time – sin +  cos  = 0
 tan  = . or  = tan–1 (1/ 3 ) = 30° Substituting
DPP NO. - 22 ; Fmin = 12.5 kg f

1. QAB =  UAB + W AB 4. Change in momentum = Impulse



W AB = 0 P  Jx î  Jy ĵ  Jzk̂


UAB = nR T 1
2 = 30(0.1) î + (80) (0.1) ĵ + (–50) × (0.1) k̂
2

 ( PV) = 3 î  4 ĵ  5k̂
2 

5  m
UAB = ( PV) | P | = 5 2 kg
2  sec .

6. Plotting velocity v against time t, we get

QAB = 2.5 P0 V0
Process BC
QBC =  UBC + W BC Area under the v–t curve gives distance.
QBC = 0 + 2P0 V0 n 2
1 1
= 1.4 P0 V0 Distance = ×2×2+ × 2 × 2 = 4m
Qnet = QAB + QBC = 3.9 P0 V0 2 2

DPPS FILE # 186

HAVE CONTROL ¾® HAVE PATIENCE ¾® HAVE CONFIDENCE Þ 100% SUCCESS


7. (C)   T ; and as there is no change in length 10. By angular momentum conservation ;

R
1 L =    mv + mvR = 2mR2
 2
T
3
mvR = 2mR2
' T 2

 T'
3v
=
 4R
 T' = T
'
T’ = (2)2 T = 4T.
Hence (C).

N
Position 2 


v u
O mg cos
8. Principle axis 2 mg
u v 1

Position 1 Also at the time of contact ;

mv 2
mgcos – N =
v 1 u 2 R
For first & second position = , =
u O v O
mv 2
v 2 1  N = mg cos –
 =  = 4.84 R
u2 2
when it ascends  decreases so cos increases
v and v decreases.
  2.2 and v + u = 96  v = 66 , u = 30
u mv 2
 mgcos is increasing and is decreasing
O R
v 11
 2 .2 
2 = u 5
 A is True  we can say N increases as wheel ascends.

Distance between two position of lens = v - u


= 36 cm DPP NO. - 23
 B is True
1.  = 2 = 3m
uv 66  30
Focal length of lens f =  = 20.63 Equation of standing wave
u  v 66  30
y = 2A sin kx cos t
 C is False y = A as amplitude is 2A.
Distance of lens from shorter image = u = 30 cm A = 2A sin kx
 D is True
2 
x=
 6
V  Vs 332  32
9. ' = = = 0.3 m
f 1000 1
 x1 = m
4
( V  V0 ) 332  64
f' = f V  V = 1000 × = 1320 Hz
s 332  32 2 5
and .x=
 6
V  V0
'' = = 0.2 m.  x 2 = 1.25 m  x 2 – x 1 = 1m
f'

DPPS FILE # 187

HAVE CONTROL ¾® HAVE PATIENCE ¾® HAVE CONFIDENCE Þ 100% SUCCESS


1 A
2. tan = , gcos = at
2 normal i normal
60°
P
20 45° 45°4
200 20 5°
10 × = = m/s2. 30°
2 2 100  400 5
10  20 incident
ray B C
Q
3. Since time of flight depends only on vertical
component of velocity and acceleration . Hence R
time of flight is
5. One can create a mental view of distribution of charge
2uy i.e. how much charge is nearer and how much is
T= where ux = cos and uy = u sin comparatively farther away.
g
In horizontal (x) direction
d = uxt + ½ gt2
2
 2u sin   1  2u sin  
= u cos  g  + g   6.
  2  g 

2 u2
= (sin cos + sin2) As a rod AB moves, the point ‘P’ will always lie on
g
the circle.
We want to maximise f() = cos sin + sin2  its velocity will be along the circle as shown by
 f ’() = – sin2+ cos 2+ 2 sin cos = 0 ‘VP’ in the figure. If the point P has to lie on
 cos2 + sin2= 0  tan2 = –1 the rod ‘AB’ also then it should have component in ‘x’
direction as ‘V’.
3 3
or 2 = or = = 67.5°  VP sin  = V  VP = V cosec 
4 8
x 1 3R 3
Alternate : here cos = = . =
R R 5 5
As shown in figure, the net acceleration of projectile
makes on angle 45° with horizontal. For maximum 4 5
range on horizontal plane, the angle of projection  sin =  cosec  =
5 4
should be along angle bisector of horizontal and
opposite direction of net acceleration of projectile. 5
 VP = V ...Ans.
4

VP 5V
Sol. (b) = =
R 4R

ALTERNATIVE SOLUTION :
Sol. (a) Let ‘P’ have coordinate (x, y)
x = R cos , y = R sin .
dx d
VX = = – R sin  =V
dt dt
135
 = = 67.5°
2 d V
 = and
dt R sin 
4. Let angle of incidence i for which deviation due to d  V 
VY = R cos  = R cos     = – V cot 
first prism is minimum, then sini = n sin30° or i = dt  R sin  
45°.
The net deviation shall be minimum if deviation due  VP = Vx2  Vy2 = V 2  V 2 cot 2 
to each prism is minimum. From the ray diagram
= V cosec  ...Ans.
in figure, it is clear that angle between AC and PQ
for net deviation to be minimum is 90°. VP 5V
Sol. (b)  = =
R 4R

DPPS FILE # 188

HAVE CONTROL ¾® HAVE PATIENCE ¾® HAVE CONFIDENCE Þ 100% SUCCESS


7. (A) Electric field at r = R 10. (A) p (B) q (C) p,r (D) q,s
(A) If velocity of block A is zero, from conservation
of momentum, speed of block B is 2u. Then K.E.
1
of block B = m(2u)2 = 2mu2 is greater than net
2
mechanical energy of system. Since this is not
possible, velocity of A can never be zero.
(B) Since initial velocity of B is zero, it shall be
zero for many other instants of time.
(C) Since momentum of system is non-zero, K.E.
KQ of system cannot be zero. Also KE of system is
E= minimum at maximum extension of spring.
R2
(D) The potential energy of spring shall be zero
where Q = Total charge within the nucleus = Ze whenever it comes to natural length. Also P.E. of
KZe spring is maximum at maximum extension of
So E = spring.
R2
So electric field is independent of a
DPP NO. - 24
2
8. Q =  r 4  r dr 1. (B) To an observer who starts falling freely under
gravity from rest at the instant stones are projected,
d 
for a = 0,  r the motion of stone A and B is seen as
R R r
dx
d =u .......(1)
 r  (R  r ) dt
R
d
=u .......(2)
dt

R
d
or, Q= R (R – r) 4 r2 dr
0  x =  and  BOA = 60°
R R
4 d  2 3
 4d  R 4 R 4   Vob 
 R r dr  r dr 
= R
 0
 
= R  3  4 
  2.  = 0 1  V 

0  sound 
Vob
 dR 3 
   1 V (straight line) ;
= 0 sound
3
Vob
 dR 3 3Ze
when V

= 0 ;  = 1.
 Q = Ze = or d =
3 R 3 sound 0

Vob 
9. From the formula of uniformly (volume) charged and as V 1  0  2
sound
solid sphere
r 3. Distance travelled by
E = 3
0 1
A = s = 10 × 2 + × 10 × 8 = 60
2
Distance travelled by
1
B = s = 10 × 3 + × 10 × 4 = 50
2
For E  r,  should be constant throughout the 60
volume of nucleus Average speed of A = = 7.5
8
This will be possible only when a = R.
50
Average speed of B = = 7.1
7

DPPS FILE # 189

HAVE CONTROL ¾® HAVE PATIENCE ¾® HAVE CONFIDENCE Þ 100% SUCCESS


4. Taking C as origin and x & y–axes as shown in 7. T cos30º + N sin30º = mg
figure.
Due to symmetry about y–axis
 3 T + N = 2 mg ..............(i)
x cm = 0
mv 2
T sin30º – N cos30º =
( 3 / 2)

 T sin30º – 3N = 4mv2 3T – 3N
= 4mv2 ..............(ii)

m1y1  m2 y 2
 
ycm =  m  m 

 1 2 

 (6 )2   4(6 ) 


   – [ (2)2 (8)]
 2   3 
 
= by (i),)(ii)
(6 )2
 (2)2
2
2mg  4mv 2
 N= ;
(m  Area) 4

8(18 2  4) 6mg  4mv 2


= 2 = 8 cm. T=
(18  4)
4 3
for N > 0
 v < 5 m/s

38
at v = 2 ij T= N ; N = 2N.
5. 3
8. 8
The slab does not contribute to deviation. For the given case Ray diagram will be as given
For minimum deviation by prism, here
r1 = r2 = 30° as shown in figure.
 sini = 2 sin 30°or i = 45°
 Minimum deviation = 2i – A = 90° – 60° = 30°

6. Amplitude phasor diagram :

Here say ER will be seen by observer which appear


to becoming from point A. To find x, the distance
of object from A we reverse the light rays by
considering ER as incident ray & find the position
of image after two refractions.
For I refraction we use
1 4 1 43
 3 =
v 2R R
 v = – 3R
  For II refraction we use
4 1 4 1
3 3
 =
x R R
 x = – 2R
 resultant amplitude = 6 2 . Hence OA = 2× 4 = 8 cm.

DPPS FILE # 190

HAVE CONTROL ¾® HAVE PATIENCE ¾® HAVE CONFIDENCE Þ 100% SUCCESS


9. Let the cube dips further by y cm and water level 2. The magnitude of the electric field is maximum where
rises by 2 mm. the equipotentials are close together. The direction
of the field is from high potential to low potential.

4. From given graphs :

3 3 
ax = t and ay =   t  1
4  4 

3 2
 vx = t +C
8
At t = 0 : vx = – 3
C=–3
3 2
 vx = t –3
Then equating the volumes 8
(/// volume = \\\ volume in figure)
 volume of water raised 3 2 
 dx =  t  3  dt .... (1)
= volume of extra depth of wood  8 

2 2 Similarly
 100 y = (1500 – 100) = 1400 ×
10 10  3 2 
dy =   t  t  4  dt .... (2)
= 280  8 
 y = 2.8 cm
 Extra upthrust As dw = F. ds = F.( dx î  dy ĵ )
water × (2.8 + 0.2) × 100 g = mg W 4
3 3    3   3  
 m = 300 gm.   dw   t î   t  1 ĵ  .  t 2  3  î    t 2  t  4  ĵ  dt
4 4    8   8  
0
m = 300 gm. ....Ans. 0

 W = 10 J
10. A – p,r,t Alternate Solution :
F = 0 Area of the graph ;
So, linear momentum conservation and centre of mass dt = 6 = V( x ) f  ( 3)  V(x)f = 3.
will not move. a x

B – q,s
and a y dt = –10 = V( y ) f  ( 4) V(y)f
So, linear momentum will not be conserved and cen-
tre of mass will accelerate W ext = E. = – 6.
C – p,s,t Now work done = KE = 10 J
D – p,s,t

DPP NO. - 25
5.
2 2
1 1  0K Q
1. U =  E2 =
2 0 2 r4
In diagram angle of emergence = 60°
KQ   BOC = 60°
V=
r  r + r = 60°
 r = 30°
1 Q2
 0K 2 4 sin 60 
U 2 r 1 0  =  = 3
= = sin 30
V2 K 2Q2 2 r2
r2
V0
U 1 6.
V0 b (a)
because 2  2 nV0
V r
so the correct option is B.

DPPS FILE # 191

HAVE CONTROL ¾® HAVE PATIENCE ¾® HAVE CONFIDENCE Þ 100% SUCCESS


8. Using equation of continuity A1v1 = A2v2
V0 (12 cm2)vA = (6 cm2) (8.0 m/s)
vA = 4.0 m/s
V0 b (b)
nV0
9. Applying Bernoulli's principle between point A and C
that are at same horizontal level
If V0 be the flow velocity of the river, then velocity of
boat relative to water = nV0. 1 2 1 2
.VA  p A = .VC  p atm
If the boat has to adopt the shortest path, then direc- 2 2
tion of velocity of boat relative to water should make
an angle greater than 90° with the flow direction of 1
 pA = (1.01 × 105 N/m2) + × 13,600 (82 – 42)
river 2
= 4.27 × 105 N/m2
Resultant velocity of boat = (nV0 ) 2 – V02 . This
velocity can have a real value only when n > 1. If n = 10. By applying Bernoulli's equation between point B and
1, then resultant velocity = 0. So the boat will follow C and using equation of continuity
the shortest path only when n > 1. So options (b) and vB = 8.57 m/s
(c) are correct. and pB = 3.70 × 104 Pascal
If boat is moved normal to flow direction, then it will gh = 3.70 × 104

b 3.70  10 4
cross the river in a time nV , where b is the width of h= = 272 mm
0
10  13,600
the river. If n  0, the boat will cross the river. So
option (d) is also correct. DPP NO. - 26
1. V = KT + C
16 – 2
7. N = 2 , N = 8 , a = = 7 ()
2 nRT
P=
v2 = 2 (7) 8  ...........(i) V
when the direction of horizontal component of the
nRT dP nRC
force F is reversed  P=  
KT  C dT (KT  C)2
a1 = 9 m/s 2 ()
and distance covered by the block before it stops As C < 0 by diagram
16  7 dP
= s1 =  < 0 for all T
29 dT
 P continuously decreases.

2. The free body diagram of the capillary tube is as


shown in the figure. Net force F required to hold
tube is
F = force due to surface tension at cross-section
(S1 + S2) + weight of tube.

16  7 S1 T×2R
Again s2 = 8 + s1 = 8 +
29
S2 T×2 R
and a2 = 7 ()

 16  7 
v2 = 0 +2 (a2) (s 2) = 2 (7)  8  
 29  mg
Free body daigram
16 7 of capillary tube
 v= m/s
3 = (2RT + 2RT) + mg = 4RT + mg
= 4 × 2 × 10–3 × 0.1 +  × 10–3 × 10 = 10.8 mN

DPPS FILE # 192

HAVE CONTROL ¾® HAVE PATIENCE ¾® HAVE CONFIDENCE Þ 100% SUCCESS


3. From conservation of energy 5 = 4 – 2 = 7,
1 1 6 = 4 – 3 = 5
mgh = mv2 + Iw2
2 2 whic h m atc hes with the given s et of beat
1 1 2 2  2 frequencies.
mgh = mv2 +  mr  
25
2  Hence (D).
7
gh = 10
(2 r2)  = wr 7. Work done by kinetic friction may be positive when it
10gh
acts along motion of the body.
= 2 r2 Friction on rigid body rolling on inclined plane is along
7
upward because tendency of slipping is downwards.
1 2 2 2 2 mgh
KE =  mr   =
25  7
Sol. 8 to 10
The time taken to reach maximum height and
4. Let ‘v’ be the initial velocity. Tangential velocity maximum height are
remains same during collision and equal to
v cos60° = v/2 u sin  u2 sin2 
t= g and H =
Let v be the normal component of velocity after 2g
impact.
For remaining half, the time of flight is
v/2 v
In  OAB : tan 60° =  v =
2 3 2H u 2 sin 2  t
v t' = (2g) = 2g 2 =
2

 1 
 Total time of flight is t + t' = t 1  
 2

u sin   1 
T= 1  
g  2
Also horizontal range is = u cos × T

u 2 sin 2  1 
= 1  
v (v / 2 3 ) 1 2g  2
Then : e = = =
v cos 30 ( 3 v / 2) 3 Let uy and vy be initial and final vertical components
of velocity.
6. As no. of beats =   uy2 = 2gH and vy2 = 4gH
For option (A) : The frequencies are :  vy = 2 uy
 1 = 550 Hz,  2 = 552 Hz,  3 = 553 Hz,
4 = 560 Hz. Angle () final velocity makes with horizontal is
The beats produced will be : vy uy
1 = 2 – 1 = 2, tan =  2 = 2 tan
ux ux
2 = 3 – 1 = 3,
3 = 4 – 1 = 10,
4 = 3 – 2 = 1
DPP NO. - 27
5 = 4 – 2 = 8,
6 = 4 – 3 = 7 1. If M0 is molecular mass of the gas then for initial
Which doesnot matches with the given set of beat
frequencies. Hence (A) is not possible. M
condition PV = M . RT ...(1)
Similarly (B) and (C) are also not possible. 0
For option (D); frequencies were; After 2M mass has been added
1 = 550, 2 = 551, 3 = 553, 4 = 558
1 = 2 – 1 = 1, V 3M T
P . = M .R. ...(2)
2 = 3 – 1 = 3, 3 0 3
3 = 4 – 1 = 8,
By dividing (2) by (1)
4 = 3 – 2 = 2
P = 3P

DPPS FILE # 193

HAVE CONTROL ¾® HAVE PATIENCE ¾® HAVE CONFIDENCE Þ 100% SUCCESS


x

2. Snell’s law = 0 sin (90° – ) = 0 1 –  sin90°
 MR 2  MR 2 
 d  =  MR 2    
 4  2 

 x 2
1 –  = cos 
 MR 2  MR 


d R 3
 = 4   = mv. = MR2 
 2 2
 x = d(1 – cos )
v
=
3. The electric field at P shall be zero if q = Q. 3R

7. As speed of ball is variable, so motion is non uniform


4. All energy is transfered to other particles.
circular motion.
5. (A) Absolute velocity of ball = 40 m/s (upwards) 8. At the highest position of ball, net tangential force is
hmax = hi = ff zero, hence tangential acceleration of ball is zero,
( 40 )2
= 10 + 9. Tension in the string is minimum when ball is at the
2  10
highest position. By conservation of energy
h = 90 m
1 1
mv2 + mg (2) = m(20 g)
(30 )2 2 2
(B) Maximum height from left = = 45 m
2  10 2
v = 16 g where v is the velocity of ball at the highest
(C) The ball unless meet the elevator again when dis- point.
placement of ball = displacement of lift
mv 2
1 So T + mg =
40 t – × 10 × t2 = 10 × t  t = 6s. 
2
(D) Let t0 be the total time taken by the ball to reach m 16 g
T= – mg = 15 mg

1
the ground then – 10 = 40 × to – × 10 × to2
2
10. (A) p,r (B) q,s (C) p,r (D) q,s
 t0 = 8.24 s. (A) The fundamental frequency in the string,
 time taken by the ball for each the ground after
crossing the elevator = t0 – t = 2.24 s. T/ 102.4 1
f0 =  3
 Hz = 320 Hz.
2 1 10 2  0 .5
Other possible resonance frequencies are f A and
f 0 = 320 Hz, 640 Hz, 960 Hz.
6. (B) The fundamental frequency in the string.

T/ 320
f0 =  = 160 Hz.
4 4  0 .5
Other possible resonance frequencies are
f B = 160 Hz, 480 Hz, 800 Hz.
(C) The fundamental frequency in both ends open
organ pipe is
Let velocity of COM after collision is v & angular
velocity is . v 320
f0 =  = 320 Hz.
conserving linear momentum 2 2  0 . 5
v Other possible resonance frequencies are
mv = 2mv  v = ............(1) f c = 320 Hz, 640 Hz, 960 Hz
2
(D) The fundamental frequency in one end open
conserving angular momentum about COM
organ pipe is
R v 320
mv. =  f0 =  = 160 Hz.
2 4 4  0. 5
= (Ring COM + mass) Other possible resonance frequencies are
f D = 160 Hz, 480 Hz, 800 Hz.

DPPS FILE # 194

HAVE CONTROL ¾® HAVE PATIENCE ¾® HAVE CONFIDENCE Þ 100% SUCCESS


DPP NO. - 28 1
5. From graph (1) : vy = 0 att = sec.
2
1. for object O1 O1 i.e., time taken to reach maximum height H is
1 1 1
  .... (1) uy 1
v 20 f
t= =
g 2
for object O2 O2
1 1 1  uy = 5 m/s . ..Ans.(i)
  .... (2) from graph (2) : vy = 0 at x = 2m
v 40 f
i.e., when the particle is at maximum height, its dis-
placement along horizontal x = 2 m
x = ux × t

1
 2 = ux ×
2
 ux = 4 m/s ....Ans (ii)
from equation 1 and 2 we get
f = 80/3
6. 4 py  y2 + 4 x2)1/2
2. As the ref lec tor appr oac hes OS, the beat
dE
frequency will decrease to zero. After this, the Force = p
dx
reflector moves away from OS thereby increasing
the beat frequency but after a long time the beat {y is not changing since p is directed along x axis}
frequency will to become constant. Hence the
correct option is (D). = p  [ 4 y 2 î  8 xy ĵ ] 

3. Equation of process = p4y | [ y î  2x ĵ ] |

P2
 = constant = C .... (1) = 4py y 2  4x 2

Ans. 4 py  y2 + 4 x2)1/2
P R
Equation of State  T .... (2)
 M Sol.7 to 9.
From 1 and 2 FBD of rod and cylinder is as shown.
PT = constant
 C is false, D is true.

As -changes to
2

P
 P changes to from equation (1)
2
 A is false.
Hence T changes to 2T .  B is true.

4. w.r.t.

Net torque on rod about hinge 'O' = 0

L
 N1 × L = mg ×
2

DPPS FILE # 195

HAVE CONTROL ¾® HAVE PATIENCE ¾® HAVE CONFIDENCE Þ 100% SUCCESS


2. (B) The line of impact for duration of collision is
mg
or N1  parallel to x-axis.
2 The situation of striker and coin just before the
Net torque on cylinder about its centre C is zero. collision is given as
 f 1R = f 2 R

or f1  f2 coin striker line of impact


Net torque on cylinder about hinge O is zero.
 N2 × L = N1 × L + mgL u
3 mg 3
or N2 =
2
Figure (A)
before collision
10. (A) p (B) q,s (C) q,s (D) q,s
(A) Work done by an ideal gas during free expansion
is zero.
(B) The angle between normal reaction on block and coin striker
velocity of block is acute (whether the block moves
up or down the incline). Hence work done by this u
force is non-zero and positive.
(C) Net electrostatic potential energy 3
rest
Figure (B)
Q2 Q2 Q2
= S1 + S2 + M12 =   after collision
8 0 a 8 0 a 4 0 b

Because masses of coin and striker are same,


Q2 Q2
=  = non-zero and positive. their components of velocities along line of impact
4 0 a 4 0 b shall exchange. Hence the striker comes to rest
( b > > a) and the x-y component of velocities of coin are u
(D) The kinetic energy of cylinder is increasing and and 3 m/s as shown in figure.
work is done on cylinder by only force of friction.
Therefore work done by force of friction on cylinder is coin
non-zero and positive. y
u P

3
DPP NO. - 29 6

1. The free body diagram of cylinder is as shown. x


O 4
Since net acceleration of cylinder is horizontal,
NAB cos30° = mg For coin to enter hole,
its velocity must be along PO
6 3
 tan  = = or
4 u
u = 2 m/s Ans. (2, 0)

3. For no ray to emerge out of side PR


2 A
or NAB = mg .... (1) A > 2C  sin > sinC
3 2

and NBC – NAB sin30° = ma A 3


 sin >
or NBC = ma + NAB sin 30° .... (2) 2 2
Hence NAB remains constant and NBC increases with or A > 120°
increase in a.

DPPS FILE # 196

HAVE CONTROL ¾® HAVE PATIENCE ¾® HAVE CONFIDENCE Þ 100% SUCCESS


4. gh r2 = 2rS cos
DPP NO. - 30
2S cos  2  1 0. 5
r= = = 10–6 m
gh 10 3  10  10
1. Flux will be maximum when maximum length of ring
is inside the sphere.
5. F = f 2  (mg)2

f (f = m2r)

F This will occur when the chord AB is maximum. Now


maximum length of chord AB = diameter of sphere.
mg In this case the arc of ring inside the sphere sub-
Now when the angular speed of the rod is 
increasing at const. rate the resultant force tends an angle of at the centre of ring.
 3
will be more inclined towards f .

Hence the angle between F and horizontal plane R
 charge on this arc = .
decreases 3
so as with the rod.
R
6. Consider a spherical shell of radius r(r > R) and  R
3
thickness dr. Then potential at centre due to it,  = = 3
0 0
r 
K dq K (C / r 3 ) 4  r 2
dV = =  dr
r r
r R 2. The change in length of rod due to increase in
r  temperature in absence of walls is
1
= (const.)  dr  = T= 1000 × 10–4 × 20 mm = 2 mm
r R r2
But the rod can expand upto 1001 mm only.
 1 At that temperature its natural length is
= (const.)  
R  = 1002 mm.
 compression = 1mm
7. Using gauss theorem
 1
  qin  mechanical stress = Y = 10111 ×
 E . ds =  0  1000

Rx = 108 N/m 2


C 2
 4  r dr
E × 4  x2 =   dv r R r3
 12 16
0 0 3. v1 = 4 cos 53º î + 4 sin 53º ĵ = î + ĵ
5 5
(C ) x
E × 4  x 2 = n 
0 x 2 R 12 9
v2 = 3 cos 37º î + 3 sin 37º ĵ = î + ĵ
5 5
(C 4  ) x
 E= n 
0 R  7
v12 = ĵ = 1.4 ĵ
5
Relative velocity in horizontal direction is zero.

4. 0.3 m
8. (Range)1 = (Range)2

2  0.1 2  0.2
2g( – 0.1)  2g( – 0.2)
Electric field will be radially outwards. g g
Electric potential decreases as we move in the
direction of electric field.  = 0.3

DPPS FILE # 197

HAVE CONTROL ¾® HAVE PATIENCE ¾® HAVE CONFIDENCE Þ 100% SUCCESS


5. (a) 4.5 m/s (b) 1.5 m/s (c) 3.75 cm 7. Since integral number of waves shall cross a point
is 5 seconds, therefore power transmitted in 5
seconds is
= <P> × 5 = 22 f 2 A2  v × 5
= 2 × 2 × (50)2 × (2 × 10–3)2 × (0.01) × 200 × 5
2
=
5

8. The equation of waves is


y = A sin(kx – t + 0)
2 
By conservation of angular momentum  where K =  ,  = 2f = 100  and A = 2
 2
0.5 × V × 0.4 = 0.5 × 4 × 1.2 at x = 2 and t = 2 y = 1 mm
v = 3u  1 = 2 sin( – 200 + 0)
also by energy conservation solving 0 = –30°
x
 y = 2 sin  
 100 t  30  
1 1 1  2 
 0.5  v 2   100  (0.3)2   0.5  u 2
2 2 2
DPP NO. - 31
v 2 9 u2
  
4 2 4 1. The velocity of profile of each elementary section
of the pulse is shown in figure 1 and figure 2.
9u 2 9 u2
  
4 2 4 pulse moving
towards right
elementary
pulse moving
8u 2 9 section
towards left
 
4 2 velocity
vector of
elementary
section
9 3
 u= 
4 2 Velocity Velocity
profile profile
So v = 3u (Figure-1) (Figure-2)

= 3 × 1.5
When both the pulses completely overlaps, the
= 4.5 velocity profiles of both the pulses in overlap region
are identical. By superposition, velocity of each
u2 elementary section doubles. Therefore K.E. of
(c) an =
r each section becomes four times. Hence the K.E.
in the complete width of overlap becomes four
times, i.e., 4k.
u2  K 100  0.3 
 r=  an  h   60m / s 2 
an  m 0.5 
N2 cos 60°
(1.5) 2
2.25 N1 cos 30°
 r= =  0.0375 m N1 N2
( 60) 60
2. N sin 30° 30° 60°
= 3.75 cm 1
N2 sin 60°

6.  = 4m and f = 50 Hz. 2 1
60° 30°
 V = f = 200 m/s
T N1 sin 30° = N2 sin 60°
 V=  N1 cos 30° + N2 cos 60° = mg
 T = v2 = (0.1) × (200)2 Solving above equation
= 400 N mg 10  10
N2 =   50
2 2

DPPS FILE # 198

HAVE CONTROL ¾® HAVE PATIENCE ¾® HAVE CONFIDENCE Þ 100% SUCCESS


3. (i) (a) The charge on the outer most surface will be
(qa + qb) and it will be uniformly distributed
K(qa  qb )
 v= Ans. ;
r
K ( q a  qb )
E= Ans.
r2
1
where K = 4  d2 y
0
aN =
(b) At a point inside the cavity of radius ‘b’ the dt 2
potential will be due to qb, – qb induced on its inner
2 x dx 2y dy
surface and due to (qa + qb) on the outer surface of  0
the sphere. a 2 dt b 2 dt

Kqb Kqb K(qa  qb ) 2vx 2 y  dy 


v= – + Ans. 2
  0
r b R a b 2  dt 
and E at that point will be only due to qb (which is 2
2v dx 2  dy  2 y  d2 y 
placed at B)     0
a 2 dt b 2  dt  b 2  dt 2 
Kqb
E= Ans.
r2 dy
[ v = const. along x-axis only = 0]
dt
qa  qb  qa  qb
(ii) R = , a = , b = Ans.
4R 2
4 a 2
4 b 2 2v 2 2(b)  d2 y  bv 2
  aN = 
(iii) 0 Ans. a2 b 2  dt 2  a2

4. (a) Parabola y = ax2 is shown. It is clear from diagram v2 a2


R=  =
that at x = 0 velocity is along x-axis and constant aN aN b
is along y-axis. So,
7. The difference in K.E. at positions A and B is

B
2
VB
2
O L gsin
VA
L
A
gsin
d2 y
aN =
dt 2 1 1
KA – KB = mv 2A  mv B2 = mg (2L cos)
2 2
dy dx = 2mgLcos .... (1) Ans.
= 2a × = 2aVx
dt dt mv 2A
TA = + mg cos
L
2 2
d y dx d x mv B2
2 = 2av = 2av2 ( 2
0) TB = – mg cos
dt dt dt L

aN = 2av2 mv 2A  mv B2
 TA – TB = + 2 mg cos .... (2)
L
2
v 1
R= = . from equation (1) and (2)
2av 2 2a
T A – T B = 6 mg cos Ans.
2 2 The component of accelerations of ball at A and B
x  y 
(b)     2   1 are as shown in figure.
a
  b 
2
 v2 v2 
Here again at x = 0 particle is at (0, ± b) moving along
 
 | a A  aB | (2g sin )2   A  B 
 L L 
positive or negative x-axis hence aN is along y-axis 
only. = 4g sin 2   16 g2 cos 2  = g 4  12 cos2  Ans.

DPPS FILE # 199

HAVE CONTROL ¾® HAVE PATIENCE ¾® HAVE CONFIDENCE Þ 100% SUCCESS


8. (A) p,q (B) p,q (C) q,r (D) q,r 5. Let the charge on intermediate shell be q (after
Sol. In all cases speed of balls after collision will be same. earthing)
In case of elastic collision speed of both balls after Potential of the intermediate shell = 0
collision will be u, otherwise it will be less than u.
KQ Kq K 2Q
 + - =0
DPP NO. - 32 2R 2R 3R

1. At the instant 3m is about to slip, tension Q q 2Q


+ – =0
in all the strings are as shown 2 2 3
T
 2Q Q   4Q  3Q 
q =   2 =  2
6
60°  3 2  
3 mg
Q
= , which is same as that before earthing
mg 3
 No charge will flow.
 3 mg = T cos 60° .... (1)
and mg = T sin 60° .... (2)
6. 2
1
 = dM
3 3  = Kx =
dx
3 M 
2M
2. | P x | = mv sin 60° = mv
2 dM =  Kx dx and K =
 2
O O
mv 3
| P y | = + mv = mv
2 2
t
F

F dx F
V= = = x dx =
 Kx dt 
O
K  dt
o

9 3
 | Pnet |  Px2  Py2 =    mv
4 3 K 4 3 2m
4 4 . .
 t= =
9 f 9F  2
| P net |  3 mv
8M 8  45  1.5
 Since, dm  A ( v dt )   = = = 2.
  9F 9  15
  dm  A  v 
 dt  Sol. 7 to 9.

 dm 
 | Fnet |  3  .v = 3  A v 2 Ans.
 dt 

2T
3. P0 + 1gh – 2gh + = P0
r
r
T= ( – 1) gh
2 2
The FBD of A and B are
Applying Newton's second law to block A
and B along normal to inclined surface
NB – mg cos 53° = ma sin 53°
mg cos 37° – NA = ma sin 37°
4.
m m
Solving NA = (4g – 3a) and NB = (3g + 4a)
5 5
GMm  – GMm  For NA to be non zero
1
KE = V1 – V2 m2 = –  4g – 3a > 0
2 2a  a 
4g
2 GM  1  or a <
= 1 –  3
a  2 

DPPS FILE # 200

HAVE CONTROL ¾® HAVE PATIENCE ¾® HAVE CONFIDENCE Þ 100% SUCCESS


DPP NO. - 33 5. (A)

1. The distribution of charge on the outer surface, de-


pends only on the charges outside, and it distributes
itself such that the net, electric field inside the outer
surface due to the charge on outer surface and all
the outer charges is zero. Similarly the distribution of
charge on the inner surface, depends only on the When the ball is just released, the net force on ball is
charges inside the inner surface, and it distributes W eff (= mg – buoyant force)
itself such that the net, electric field outside the inner The terminal velocity ‘vf’ of the ball is attained when
surface due to the charge on inner surface and all the net force on the ball is zero.
inner charges is zero.  Viscous force 6r vf = W eff
Also the force on charge inside the cavity is due to
the charge on the inner surface. Hence answer is 2
When the ball acquires rd of its maximum velocity vf
option (A). 3
2
2. The distance between the orbiting stars is the viscous force is = W eff.
3
d = 2rcos30° = 3 r. The net inward force on orbiting
stars is 2 1
Hence net force is W eff – W eff = W eff
3 3
Gm 2 GMm Gm 2 mv 2
cos 30    cos 30 
d2 r2 d2 r a
 required acceleration is =
3
m  4  2r 3
 G  M 
 3  T2 6. If we complete the trapezium as shown It becomes
an equilateral triangle
r3  A = 60°

or T =2 G  M  m  A

3
  A   min 
 sin  
 2   5 5
A
m d Smn
2
M
10
d r d
 60   min 
sin  
 2   2
60 , min = 30°
sin
1 1 2
3. k x 02 + Mgh = k(x0+h)2 + 0
2 2
2Mg
h= – 2x0
k
Maximum downward displacement
2Mg
=[ – 2x0 ] 7.
k

u2
4. H = 3
2a  Clearly, PM = cm
2
a is same for all the three cases.
1
(u sin  )2 u2 37º > sin–1 n  a(3 / 2)
HA = , HB = 0
2a  2a 
3 1
(u cos  )2 >
5 3a
and HC = n0 
2a  2
 HB = HA + HC

DPPS FILE # 201

HAVE CONTROL ¾® HAVE PATIENCE ¾® HAVE CONFIDENCE Þ 100% SUCCESS


9a F1=20N N=40 N=40 F2=60N
3n0 + >5 20kg 10kg
2
f=20N f=20N
9a FBD of both blocks
>1
2 Hence magnitude of friction force on both blocks
is 20 N and is directed to right for both blocks.
2
a> Normal reaction exerted by 20 kg block on 10 kg
9
block has magnitude 40 N and is directed towards
right. Net force on system of both blocks is zero.

8.
DPP NO. - 34
2. U = 3x + 4y
Fy  U / x 
2.5 ay = = =–3
 = 0.1 g / cm = 10–2 Kg/m m m
25
Fy  U / y 
Ist overtone ax = = =–4
m m
 s = 25 cm = 0.25 m

 a = 5 m/s2
1 T
fs = Let at time 't' particle crosses y-axis
s 
1
pipe in fundamental freq then –6= (– 3) t2
2
 t = 2 sec.
Along y-direction :
1
y = (– 4) (2)2 = – 8
2
 p = 160 cm = 1.6 m  particle crosses y-axis at y = – 4
At (6, 4) : U = 34 & KE = 0
V At (0, – 4) : U = – 16
fp =   KE = 50
p

 by decreasing the tension , beat freq is 1


or, mv2 = 50
decreased 2
 fs > fp  v = 10 m/s while crossing y-axis
 fs –fp = 8
3.
1 T 320
  = 8
0.25 10  2 1 .6
 T = 27.04 N

9. (A) p,s (B) p,s (C) q,s (D) r Tension in elementary section of width dx is
The minimum horizontal force required to push the T =  xg ( = mass / length)
two block system towards left  extension of length x (= BC) of wire is
= 0.2 × 20 × 10 + 0.2 × 10 × 10 = 60. x
Hence the two block system is at rest. The FBD ( x g) 2
x =  dx =  x ... (1)
of both of blocks is as shown. The friction force f YA 2YA
0
and normal reaction N for each block is as shown.
 extension in total length of wire (=AB) is 2x
F1=20N F2=60N
 2 g
2x = ... (2)
2 YA
fmax=40N fmax=

DPPS FILE # 202

HAVE CONTROL ¾® HAVE PATIENCE ¾® HAVE CONFIDENCE Þ 100% SUCCESS


 from equation (1) and (2) 7. (A) Charge distribution on system is shown
 below
x=
2
AC x
Ans. = = ( 2 – 1)
PC x

4. i = wavelength of the incident sound


u
10u  19u
= 2 =
f 2f
fi = frequency of the incident sound
10u  u So electric lines of forces are as shown below
= f = 18 f = f r = frequency of the
u 19
10u 
2
reflected sound
r = wavelength of the reflected sound
10u  u 11u 11 19 u
= fr = 18 f × 19 = .
18 f
i 19 u 18 f 9
 r = 2f 11 19u = 11 Ans. Since number of lines of force are proportional to

charge so no. of lines of forces emerging from inner
sphere should be equal to the no. of lines of forces

5. rP = ( î + ĵ ) t emerging from outer shell.

rQ = (2 î + ĵ ) + (– î + 2 ĵ )t
    6  10 9 3  10 9 3  10 9 
rQP = rQ – rP = 2 î + ĵ + (–2 î + t ĵ ) 8. VB = K  2
   = 200 V
 27  10 18  10 2 18  10 2 

rQP = (2 – 2t) î + (1 + t) ĵ
x = 2 – 2t y = 1 + t  6  10 9 3  10 9 3  10 9 
 x = 2 – 2 (y – 1) VA = K  2
  
 27  10 18  10  2 6  10 2 
x + 2y = 4 Ans.
= 200 V – 150 V + 450 V
6. Force on cone while it is penetrating the sand is shown = 500 V
in F.B.D. below VC = VB (as shell is conducting)
Applying work energy theorem to the cone as x Therefore, VC – VB = 0
changes from 0 to d VB – VA = – 300 V
KE = work done by mg + work done by
resistive force R
9. Potential at point B and point C increases by same
value, keeping their difference unchanged.

DPP NO. - 35
d
2
KFinal – KInitial = mgd –
 kx
0
dx 1. F.B.D. of man and plank are -

d
2
0 – mgh = mgd –
 kx
0
dx

kd 3
 = (mgd + gh)
3
3mg
 k= (h  d)
d3

DPPS FILE # 203

HAVE CONTROL ¾® HAVE PATIENCE ¾® HAVE CONFIDENCE Þ 100% SUCCESS


For plank be at rest, applying Newtons second law 4. At position A balloon drops first particle
to plank along the incline So, uA = 0, aA = – g, t = 3.5 sec.
Mg sin  = f ...............(1) 1 2
and applying Newton’s second law to man along SA =  gt  ...........(i)
2 
the incline. Balloon is going upward from A to B in 2 sec.so
mg sin  + f = ma ...............(2) distance travelled by balloon in 2 second.

 M  1 
a = g sin   1   down the incline  SB  a B t 2  ..........(ii)
 m  2 
Alternate Solution : aB = 0.4 m/s 2 , t = 2 sec.
If the friction force is taken up the incline on man, S1 = BC = (SB + SA) ...........(iii)
then application of Newton’s second law to man
Distance travell by second stone which is droped
and plank along incline yields. from balloon at B
f + Mg sin  = 0 ..........(1) u2 = uB = aBt = 0.4 × 2 = 0.8 m/s B

>
mg sin  – f = ma ..........(2) t = 1.5 sec. SB
Solving (1) and (2)
1

>
 
 S 2  u 2 t  gt 2  A

>
...........(iv)
 M  2 
a = g sin   1   down the incline
 m Distance between two stone SA
Alternate Solution : S = S1 – S2 .

>
Application of Newton’s seconds law to system of C
man + plank along the incline yields
F F
mg sin  + Mg sin  = ma
a
 M R 2R
a = g sin   1   down the incline 5.
 m
f
2. As ON = MN = OM = r
f = ma ...(i)
So it is equilateral triangle :
F2R – FR – fR = I ...(ii)
 Potential at N due to two dipoles ;
a = Ra ...(iii)
V = V1 + V2
a
FR – fR = I.
r r 3  R
 , ,0 
2 2 
N  
Ia
F – ma =
r r R2
 - 60°
O 60° r

 
M   I 
P (0,0,0) P (r,0,0) F =  m  2  a.
 R 

Kp cos 60 Kp cos(   60) F


= + =0 a=
r 2
r 2 m  I/R2

mF
x x rel f = ma = m  I
3.  x rel =  x R2
1 
mF
d 2 x rel d2 x
 f = m I
dt 2 dt 2 R2
arel =  g
10
f= N
3

DPPS FILE # 204

HAVE CONTROL ¾® HAVE PATIENCE ¾® HAVE CONFIDENCE Þ 100% SUCCESS


F 5
6. a = I = m 2
m 3
R2  3  2 2
(C) T = 2 mgd = 2  3g
a 5 mg
= = 2
2 6
5 5
v=0+ 3 = m  / 2 A 
6 2 (D) T = 2 Ag = 2 Ag = 2 2g
5
 = o +  t = 0 + 3 = 5
3
1 1 2 DPP NO. - 36
KE = mv2 + I
2 2 1. (C) Work done against friction must equal the initial
1 1 5 5 kinetic energy.
= 255   4 
2 2 2 2
 
1 v2 1
25 75 mv 2 =  mg dx ; = A g 2 dx ;
= 25 + = J 
2
 2
x
2 2 1 1

7. F2R – FR = I
v2

 1
= Ag  x 
FR 2  1
=
I

 FR  v2 = 2gA v = 2g A
=0+  t
 I 
2. F.B.D. for minimum speed (w.r.t. automobile)
1 2
KE = Iw
2

1  F 2 R 2  2
= 2  I  2 t
 I 

F 2R 2 100  1 3  3 25  9 :
= = = = 112.5 J.
2I 2  4 2

8. (A) p (B) q (C) p (D) s


(A) In frame of lift effective acceleration due to mv 2
g 3g
f y' = N – mg cos  – sin  = 0.
gravity is g   downwards R
2 2
2 mv 2
 T = 2 f x' = cos  + N – mg sin  = 0
3g R

k g
(B) K = mg  mv 2 mv 2
m L  cos  + (mg cos  +
constant acceleration of lift has no effect in time R R
period of oscillation. sin ) – mg sin  = 0
m  (Rg cos   Rg sin )
 T = 2 = 2  v2 =
k g (cos    sin )

for  = 45º and  = 1 :

Rg  Rg
vmin = =0
1 1

DPPS FILE # 205

HAVE CONTROL ¾® HAVE PATIENCE ¾® HAVE CONFIDENCE Þ 100% SUCCESS


5. In elastic collision the velocities are exchanged if
masses are same.
 after the collision ;
VC = 0
VA = v
Now the maximum compression will occure when
both the masses A and B move with same velocity.
 mv = (m + m) V (for system of A – B and spring)
v
V=
2
F.B.D for maximum speed (w.r.t. automobile) 2
2
1 v
mv  KE of the A – B system = × 2m  
f x' = cos  – mg sin  – (mg cos  2 2
R
mv 2
mv 2 =
+ sin ) = 0 4
R
And at the time of maximum compression ;
for  = 45º and  = 1
2
vmax =  (infinite) 1 1 v 1
mv2 = × 2m   + K X 2max
2 2 2 2

3. m
 X max = v
2K

Taking cylindrical element of radius r and thickness


dr
M
dm = × (2r  dr)
(R 22  R12 ) 

R2
2M
AB = d e = dm r 2
=  (R 2
.r 3 dr 6.
  R1 2  R12 )

1
= m (R 22  R12 ) 40  10
2 E= = 100 V/m
0 .3
Using parallel axis theorem
(near the plate the electric field has to be uniform
1  it is almost due to the plate).
IXY = m (R 22  R12 ) + MR22
2 For conducting plate

4. Let m be minimum mass of ball. E= 
0
Let mass A moves downwards by x.
From conservation of energy,   =  0E
Therefore %,  = 8.85 × 10–12 × 100
1
mgx = kx2 = 8.85 × 10–10 C/m 2
2
7. Direction of E.F. at B is towards A that will exert
 2mg  force in this direction only, causing the positive
x=  k  
  c har ge to m ove. [ E is per pendic ular to
For mass M to leave contact with ground, equipotential surface and its direction is from high
kx = Mg potential to low potential.]
 2mg 
K  = Mg 8. W = q.dV
 k 
= –1 × 10–6[20 – (–20)]
M = – 4 × 10–5 J.
m= .
2

DPPS FILE # 206

HAVE CONTROL ¾® HAVE PATIENCE ¾® HAVE CONFIDENCE Þ 100% SUCCESS


DPP NO. - 37 4. Kx = V(2000) (10) – V (1000) (10)

1. For first collision


10
v = 10 m/s. = [ 1000 × 10]
2000
 (5 )
t1 =
10
= /2 sec.
velocity of sep = e. velocity of opp.
1
v2 – v1 = (10)
2
v2 – v1 = 5 m/s
for second collision
Kx = 50 N ... (b)
2 (5)
 t2 = = 2 2
5 1  50  1 2500
Ustored = × (100)   = 
 total time t = t + t2 2  100  2 100
= /2 + 2  = 12.5 J
t = 2.5 
 Ve
5. At x = 5 cm,  x = slope of figure 1

at x = 5 = +ve
2.  Ve
So Fx = – q  x = – ve

The friction force will reduce v0 , hence translational  Vg


K.E. at y = 15 cm, = slope of figure 2
y
The friction force will increase  at y = 15 = – ve
There is no torque about the line of contact, angular  Vg
momentum will remain constant So Fy = – m = +ve
y
The frictional force will decrease the mechanical
So particle will try to move towards – x direction
energy.
and +y direction.

6. at (25, 35),

 Ve
Fx x  25 = – q x
x  25

3. Equation of the component waves are :  2  10 4 



=   0. 1
  × (20 × 10–6) = 4 N
y = A sin(t – kx) and y = A sin (t + kx) 
 
where; t – kx = constant or t + kx = cosntant
Diffeentaiting w.r.t. 't' ;  Vg
Fy  m
y  35 y
dx dx y  35
– k =0 and + k =0
dt dt  10 3 
= – (200)   = – 2N
dx    0 .10 

v = = and v = –
dt k k Fnet = 4 î  2 ĵ = (200) a

  4 2
i.e.; the speed of component waves is   . î  ĵ = a
k 200 200

Hence (B) a = (2 î  ĵ ) × 10–2 m/sec 2

DPPS FILE # 207

HAVE CONTROL ¾® HAVE PATIENCE ¾® HAVE CONFIDENCE Þ 100% SUCCESS


7. W (5, 15)  (25, 35) = U(25, 35) – U(5, 15) 3. Both blocks loose contact immediately after the
release.
= (0 + (200) (–1.5 × 10–3)) – [(20 × 10–6)
m
(1/2 × 104) + (200) (–1.5 / 10–3)] T P = 2 ,
4K
= – 0.1 J
m
8. (A) p, q, r, s,t; (B) p, q, r, s,t; (C) p, s,t; T Q = 2
K
(D) p, q, r, s,t
(p, s) since there is net impulse, translations motion   T Q = 2T P
will occurs for all cases. TQ
(r,q) only in C, impulse is passing through centre of Q comes at lowest position at time travelling
2
mass. Hence rotation will occur and angular
momentum will increase in all cases except (C). 2mg
a distance downwards.
(t) About all the points on the line of action of the K
impulse, torque is zero. Hence angular momentum
will conserve for many points. TQ
In time , i.e. time period of P (T P) the block P
2
come back to original position
DPP NO. - 38
2mg
 The distance between P and Q is
K
F
1. T = ...........(i) TQ
L 4. At t = both the blocks are at extreme position
2
dv V and their velocity is zero. [Soln. of SSI Sir]
F = A  L2 = LV
V
dx L  VP = VQ = 0
F
= ...........(ii)
LV P V P 1.5  140  10 3
6. B = – =– =–
From (i) (ii),(i) (ii) V / V V  0.2  10  3
F  = 1.05 × 109 Pa .
   [V].
 Ans. 1.05 × 109 Pa.

0 
m 2 7. Moment of inertia of one rod about the axis of frame
2. – T 
   dx x
T 0 2 2
m L m L m 2  1 1
=   +   = L   
m 2x 2
 4  12 4 2 4  12 4 
 T= 
 2
m L2
=
F F 12
Y =  = Ay
A   Moment of inertia of frame = ML2/3.

9. (For the above two questions)


Newton's law applied on C.M. gives
mg sin – f = ma .... (1)
m 2 x 2 Writing  about C.M., we have
dx
 =  2 L mL2
AY f. =
 .... (2)
2 3
m 2  3 from the condition of rolling, we have
 =
 6 AY L
a=  .... (3)
2
2  3
 = from (1), (2) and (3)
6y
 = 2 2 mg sin  3
f= and a = g sin
2 = 21 5 5

DPPS FILE # 208

HAVE CONTROL ¾® HAVE PATIENCE ¾® HAVE CONFIDENCE Þ 100% SUCCESS


DPP NO. - 39
m = 1kg Q
1. From the free body diagram of the sphere :
FV = 4 mg – 2 mg – FB
4. µ = 1.5 S P R P R
 FV = 2 mg – FB
4 3 
 6  r V = r    g
3 2 
4 3 M = 11kg
(since 4m = r   )
3
If the point P has an acceleration a upwards then the
2 2 (   2 ) g
 V= r acceleration of point R will be a downwards.
9 

M = 11kg

The point R has an acceleration a downwards


so the block will also have an acceleration a
downwards.

S P

2. In the figure shown M = 11kg

The point P has an acceleration a upwards, the block


has an acceleration a downwards so the acceleration
of S will be 3a downwards. (because
 
a S  aP 
 ablock ).
2
 OO P &  P are similar
The point Q will also have an acceleration 3a towards
OO OP right.

   P ..............(1)
2T T
also   OO C &   C are similar
OO  OC a
= .............(2) The F.B.D. of 11kg block
  C
By equation (1) and (2) 110 N
OP OC OP P
=   Ans.
P C OC C
3a
3. As wave has been reflected from a rarer medium, The F.B.D. of 1kg block
T
therefore there is no change in phase. Hence equation 15N
for the opposite direction can be written as
y = 0.5A sin (–kx – t + ) Using FBD of 11 kg block, which will have acceleration
= – 0.5A sin (kx + t – ) a downwards.

DPPS FILE # 209

HAVE CONTROL ¾® HAVE PATIENCE ¾® HAVE CONFIDENCE Þ 100% SUCCESS


110 – 3T = 11a ........ (1) (in downwards direction) 9. (i) Cons. linear momentum
For 1 kg block, which will have acceleration 3a,
T – 15 = 3a (in horizontal direction)
or 3T – 45 = 9a ............. (2)
on adding equation (1) & (2) we get
20a = 65  4a = 13 m/s2

900(10  a)
5. 1.15 × 108 = – 2m.v + 2v.m = 0 = MVcm
 d2 
  Vcm = 0
 4 
  (ii) As ball sticks to Rod
Conserving angular momentum about C
6 0.06 2v.m. 2a + 2mva = 
 d= cm =
10  10 

6  10 2
m= m
10 

6  10 2  8m. 36a 2 
Ans.
10 
m 
=   2m. a 2  m. 4a 2 
 12 

6mv.a = 30 ma2.
6. The situation is shown in figure.
(a) From figure h =  (cos  – cos 0) v
and 2 = 2gh   =
5a
= 2g (cos  – cos 0) ....... (1)
2
Again T – mg cos  = m /  ....... (2) v2
1 1
Substitting the value of 2 from eq. (1) in eq. (2) (iii) KE = 2 = . 30 ma2 ×
2 2 25a 2
we get

3mv 2
= .
5
 
 T
h DPP NO. - 40

mg 1. Upward force by capillary tube on top surface of
liquid is
T – mg cos  = m {2g (cos  – cos 0) /} fup= 4a cos 
If liquid is raised to a height h then we use
or T = mg cos  + 2mg (cos  – cos 0)
4a cos = ha2 g
or T = mg (3 cos  – 2 cos 0)
or T = 40g (3 cos  – 2 cos 0) newton 4 cos 
or h = ag Ans.
Ans. T = 40 (3 cos  – 2 cos 0) kg f.

(b) Let 0 be the maximum amplitude. The maximum


tension T will be at mean position where  = 0.
F
 Tmax = 40 (3 – 2 cos 0) 2.  =   

r2y r2
But Tmax = 80
Solving we get 0 = 60° Only option 'radius 3mm, length 2m' is satisfying
Ans. 0 = 60° the above relation.

DPPS FILE # 210

HAVE CONTROL ¾® HAVE PATIENCE ¾® HAVE CONFIDENCE Þ 100% SUCCESS


0.5 3.2 gm 3.2  10 3 3 .2 32
3. Velocity gradient = 2.5  10  2 6. µ = =  2 = 40 = 4000 kg/m
2 40 cm 40  10
as force on the plate due to viscocity is from upper
as well as lower portion of the oil, equal from each =

part, 2

dv   = 2 ...........(1)
Then, F = 2 A
dz
0.5 v 1 T
= 2 ×  × (0.5) 2
f= =
1.25  10  2 µ

1000 1 T
 =
  = 2.5 × 10–2 kg – sec/m 2
64 2  40  10  2 32 / 4000

2
1000 2  32
  64  2  40  10  4000
=T
 

1000 32
× =T
64 4000
4.
10
T = N
8

10 / 8
5 5 2 cm
sin i = 1 sin e 10  6
2

2 2
4cm  h 2 10 7 40
vc y = .05  10  2 =
8 (.05)
4 cm 40  10  2
=1×  h = 4 cm
16cm2  h2 = 109 N/m2. [ Ans. 1  109 N/m2 ]

7. Torque of friction about A is zero.

8. Angular momentum conservation about point A.


5. Lin = mv0r – mk 20
Lfin = 0
Lfin = Lin
 v0 = 0k 2/r.

From conservation of energy 9. acm = –g

1 v 02
mgh = mv2 02 = v02 – 2gs S=
2 2g
1
 mg sin = mv2
2
DPP NO. - 41
v2
 2g sin = = aC
 1. (B) VB > VD = In a Wheatstone's bridge circuit shown
g cos = at if PS = QR, VB = VD . No current flows between B
and D.
Total acceleration a = a c2  a 2t
If PS < QR , VB > VD current flows from B to D.
= g cos 2   (2 sin )2 = g 3 sin2   1
If PS > QR , VB < VD current flows from D to B.

DPPS FILE # 211

HAVE CONTROL ¾® HAVE PATIENCE ¾® HAVE CONFIDENCE Þ 100% SUCCESS


Alternate : VB = VC
B VD – VC = i3S = positive
 VD > VC
P Q
 VD > VB
A C If resistance R is zero and all other resistances are
G non–zero, then PS > QR
and similarly, we get VD > VB
R S Hence if PS < QR, VB > VD .
D

Let resistance P = 0 and all other resistances Q,R,S,G


are non–zero then PS < QR condition is satisfied.

0 0
P= P=
2.
G
 i1

VA = VB 1 1
 VA – i1R = VD K.E. system = × 2 mR 2 02 + m(2R) 20 2 +
2 2
 VA – VD = i1R = positive (or current flows from A to
D through G, then VA > VD) 1 1
 VA > VD m( 2 R)2 r2 + 2m( 2 R)2 02
2 2
 VB > VD
= 6 mv02
Let resistance S = 0 and all other resistances P,Q,R,G
are non–zero then PS < QR condition is also satis- 1
K.E.2m = × 2m ( 2 R)22 = 2mv02 .
fied. 2

B B
i2
P Q P Q
2T
3. a =
G G m
A C A C
v = u + at  0 = u – at
0 
R S= R
S=
0
u um
D D t= =
a 2T

um
Total time T = 2t = .
T
VD = VC
 VB – i2Q = VC = VD
 VB – VD = i2Q = positive 4. According to given condition,
 VB > VD
Suppose resistance Q is zero and all other resis- 13 v 7v
=
tances P,R,S,G are non–zero then PS > QR. 
4   e
  
2   2e 
 2   2 
B B
Q=0 Q=0

P P e=
24
A G C A C
G 10e
R  R So, r =
S S 6
i3
D D
5
r=
72

DPPS FILE # 212

HAVE CONTROL ¾® HAVE PATIENCE ¾® HAVE CONFIDENCE Þ 100% SUCCESS


5. For TIR at B, the angle of incidence i  c 8. The energy of any geostationary satellite is the
sum of kinetic energy of satellite, interaction energy
of satellite and its own planet and interaction
energy of satellite and star. Both planets have
same mass and same length of day. Geostationary
satellite - planet system will have same interaction
energy in either planet. Also kinetic energy of both
satellites will be same. But the satellite-sun system
will account for the energy difference.

GMm 0
Ui = – + Usatellite – planet
2r
& r + i = 90  i = 90 – r
by snell’s law at pt A, GMm 0
sin 45° = n sin r = n cos i Uf = – + Usatellite – planet
2 ( 4r )
Now  i > c  sin i > sin c
3 GMm 0
1 Emin = Uf – Ui =
 cos r > 8r
n
DPP NO. - 42
1
 n>
cos r
1. In a binary star system
1 1 = 2
 n>
2
1  sin r

2. Strain () = = T = (10–5) (200)
1 
 n>
1 = 2 × 10–3
1
2n 2 Stress = Y (strain)
Stress = 1011 × 2 × 10–3 = 2 × 108 N/m2
 Required force = stress × Area
2n
 n> = (2 × 108) (2 × 10–6) = 4 × 102 = 400 N
2n2  1
400
 Mass to be attached = g = 40 kg
3
 2n2 – 1 > 2  n >
2 3. [Ans. v 1 =30, v 2 = – 25, v 3 = -35/3, v 4 = -25 cm]

6. The time in which the planet rotates about its axis 4. Ans. 64
is not given for either planet.
p2
=
v
7. For geostationary satellite, time period = 1 planet
2
day (by def.) 2 p2 1 v 1
Let T = 1 planet day 1 = p12 ×  2 v 2
T 0 = 1 planet year
9 1.5 400 9 3
4 2 3 4 2 3  m  =   = =
rG = r   16 3 1200 16  3  2 32
Now T2 =
Gm Gm  M 
1 6  32
6 = = 64
2 2 3
4 3
= r  T03  T = T 0
GM

DPPS FILE # 213

HAVE CONTROL ¾® HAVE PATIENCE ¾® HAVE CONFIDENCE Þ 100% SUCCESS


5. By energy conservation,

3 1 m(3 )2 2
mg. = . .
2 2 3

g
= 

and velocity of centre of mass of rod BC
 Ratio = 3.
g
Vcm = .2
 2. The only force acting on the body is the viscous force
vdv
Vcm = 2 g Here, m = –6rv
dx
If time taken by centre of mass of rod BC from break-
= – rv
ing position to line PQ is t.
0 x
1 2 mdv   rdx mv
8 =  g  t     x= .
2 v 0 r
3. The image of a point closer to the focus will be farther.
 As the transverse magnification of B will be more than
t= 4 g
A, the image of AB will be inclined to the optical axis.
 = .t = 4 radian.
F GM
4. a1 = = 2
m r
6. iR =  (58.3)
It is same in both cases
& iX =  (68.5)
a1
wher e  is the potential gr adient of the  a2 = 1
potentiometer

R 58.3 
 = 5. Loudness  = 10 log10
X 68.5 0
2
10 58.3   2 –  1 = 10log10 &
=  X = 11.75  1
X 68.5
P
 =
7. The maximum P.D. which we can measure by this 4r 
potentiometer is V
2 r12
8. Any change can be done which assures p.d. across  = 2
R or X less then or equal to V
1 r2

r2
DPP NO. - 43  ( + 20) –  = 10 log10
r22

1. Originally r
VA = VD = VE = 20 log10
r2

r
 = 10  r2 = 0.1r
r2

After connecting C & B. The equivalent circuit will be  shift = r – 0.1 r = 0.9 r.
[Now VA = VD =VE and VC = VB ]
9r
Ans.
10

DPPS FILE # 214

HAVE CONTROL ¾® HAVE PATIENCE ¾® HAVE CONFIDENCE Þ 100% SUCCESS


Sol. 6 to 8 3. a = 3t2 + 1
u  vB dv
v 1
The angular speed of rod =  = 2
/2 dt
= 3 t2 + 1   dv   (3 t  1) dt
0 0

2u 1
As given vB = 0  = Ans. v = t 3  t 0 = 2 m/s.
 

The time after which centre of rod reaches the  v = t3 + t
u s 1
highest point is t0 = 3
g   ds   ( t  t ) dt
0 0
The angular acceleration of rod is zero and in the
given time to the rod undergoes angular displacement
1 1
  S=  = 0.75
. 4 2
2
4. 8000
 from  = t
Conserving angular momentum m.(V1 cos60°).
 2u u gL V2
 =   g or u = 4R = m.V2.R ; = 2.
2 4 V1
Conserving energy of the system

DPP NO. - 44 V2
m.(V1 cos60°). 4R = m.V2.R ; = 2.
V1
1. By energy conservation between A & B
G Mm 1 GMm 1
  mV12 =   mV22
2R MgR 1 4R 2 R 2
 Mg +0= + MV2
5 5 2
1 2 1 2 3 GM
V – V1 =
2 2 2 4 R

1 GM
or V12 =
2 R
O
R
53º
A
37º 1 8000
R–R cos53
=2R/5 B
V1 = 64  10 6 = m/s Ans. 8000
R–R cos37= 2 2
R/5 Reference line
37º
g
g cos37
7. On the positive side of x axis, potential is zero at
distance x 1 (it is between both charges), then

k.6e k.10e
  r = 3nm
r 8 r
2gR
V= For the left side
5
k.6e k.10e y
Now, radius of curvature r

x2 8  x2
V2 2gR / 5 R
=    x 2 = 12nm R xc
ar g cos 37 2
x1  x 2 x
R= =7.5nm x2 x1
V2 2gR / 5 R 2
r=  
ar g cos 37 2 |X c| = X 2 – R = 4.5 nm

k.6e k.10e
2. Relative displacement of glass window w.r.t. cyclist Vc = –
4.5  10 9
12.5  10 9
is 20 cm time taken = 1 sec.
So, relative velocity of glass window w.r.t. cyclist = 4 4 8
= 9×109×109×1.6×10–19    = 1.44 ×
20 3 5  15
cm/sec. = 0.2 m/sec.
1 = 0.77V

DPPS FILE # 215

HAVE CONTROL ¾® HAVE PATIENCE ¾® HAVE CONFIDENCE Þ 100% SUCCESS


8. (A) p,r (B) q, r (C) q, r (D) q, r
Initially the image is formed at infinity. v 20
3. R1 = v0 v0cos
(A) As m is increased the focal length decreases. g cos 
g
Hence the object is at a distance larger than focal
length. Therefore final image is real. Also final image ( v 0 cos )2
R2 = g
becomes smaller is size in comparision to size of g
image before the change was made.
(B) If the radius of curvature is doubled, the focal R1 1
 = 8 Ans. 8
length decreases. Hence the object is at a distance R2 (cos )3
lesser than focal length. Therefore final image is
virtual. Also final image becomes smaller is size in
comparision to size of image before the change was
made.
(C) Due to insertion of slab the effective object for
lens shifts right wards. Hence final image is virtual.
4.
Also final image becomes smaller is size in
comparision to size of image before the change was
made.
(D) The object comes to centre of curvature of right
 334  9  325
spherical surface as a result. Hence the final image f' =  700 = × 700 = 650 Hz.
 334  16  350
is virtual. Also final image becomes smaller is size in
comparision to size of image before the change was
made. 5. Consider two small elements of ring having charges
+dq and – dq symmetrically located about y-axis.
The potential due to this pair at any point on y-
DPP NO. - 45 axis is zero. The sum of potential due to all such
possible pairs is zero at all points on y-axis.
1. The linear relationship between V and x is R
V = – mx + C where m and C are positive constants. Hence potential
y at P(0, ) is zero.
2
 Acceleration +dq ++++ –––
––
++
–dq
––

dV
++

a= V = – m(– mx + C) d  
++++++++

––––––

dx d

x´ x
+ –
+

––
++

+

++ –
++++ –
–––


 a = m 2x – mC
Hence the graph relating a to x is. 6. Since all charge lies in x-y plane, hence direction
of electric field at point P should be in x-y plane
Also y-axis is an equipotential (zero potential) line.
2. Relative to lift initial velocity and acceleration of coin
Hence direction of electric field at all point on y-
are 0 m/s and 1 m/s2 downward
axis should be normal to y-axis.
 The direction of electric field at P should be in
x-y plane and normal to y-axis. Hence direction of
electric field is along positive-x direction.

7. Consider two small elements of ring having charge


+dq and –dq as shown in figure.
The pair constitutes a dipole of dipole moment.
1 dp = dq 2R = ( Rd) 2R
 2= (1) t2 or t = 2 second
2 The net dipole moment of system is vector sum
of dipole moments of all such pairs of elementary
charges.

DPPS FILE # 216

HAVE CONTROL ¾® HAVE PATIENCE ¾® HAVE CONFIDENCE Þ 100% SUCCESS


By symmetry the resultant dipole moment is
DPP NO. - 46
along negative x-direction. 1. The electric field intensity due to each uniformly
charged infinite plane is uniform. The electric field
intensity at points A, B, C and D due to plane 1,
y plane 2 and both planes are given by E1, E2 and E
+++
+ ––– as shown in figure 1. Hence the electric lines of
––
++
–dq
forces are as given in figure 2.

––
++

z
++++++++

––––––
d
E – E
 1 1
x´ x E E
B E2 E2 A
+

––
++

+ – 1 x
++ +

++++
+ – E2 E2
+dq –– –– C D
y´ E E
E1 E1
2
(figure 1)
 net dipole moment
z
 / 2  / 2
2
=–  (dp cos ) î    (2 R
 / 2  / 2
cos  d) î
x
2
= – 4R  î

1  n   1 1  (figure 2)
   1   
8. (A) f n R R
s 1
2 
  Aliter : Electric lines of forces originate from
positively charged plane and terminate at negatively
f = – ve charged plane. Hence the correct representation
1 of ELOF is as shown figure 2.
P= = –ve q,t
f

1  n   1 1 
   1   
(B) f  n R1 R 2 
s   2.
 

f = +ve

1
P= = +ve p,r,s 
f Force on q = Eq = (1  cos o ) q = f
2 0

1  n   1 1 
   1   
(C) f  n R1 R 2 
s  
 

1
f=+, P= = +ve p,r,s
f

R
(D) f = Consider a ring of radius y and thickness dy. Flux
2
through this ring d =(E cos)2ydy
1
P= = –ve r,q,t 0
1 q
f 2ydy cos 
 Total flux =  4 0 r2
0

DPPS FILE # 217

HAVE CONTROL ¾® HAVE PATIENCE ¾® HAVE CONFIDENCE Þ 100% SUCCESS


7. by energy conservation
q
= 2 (1  cos  0 ) 1 1 mR v2 1
2
0  20  0.3  0.3    2  mv 2
2 2 2 R 2
f 2
mv = 1.2 J
So,  

mv 2
Translation K.E. = = 0.6 J
2

8. (A) – p,q,s, (B) – r, (C) – r,t ; (D) – p,q,s,t.


(p) Parallel beam can be obtained from concave mir-
ror and convex lens when point object is at focus.
(q) Real image for real object is for concave mirror
and convex lens.
(r) Virtual and diminished images are obtained for
convex mirror and concave lens.
dv
3. (Moderate) a = v = cx + d (s) Real and magnified image is obtained for concave
dx mirror and convex lens.
Let at x = 0 v = u (t) The direction of motion of image is in the same
direction as motion of object in lens and opposite in
v x
mirror.
  v dv   (cx  d) dx
u 0

or v2 = cx 2 + 2dx + u2
DPP NO. - 47
v shall be linear function of x if cx 2 + 2dx + u2 is
1. As seen from the figure
perfect square

4. (C)
Velocity of approach of P and O is

dx
– = v cos 60° = 5 m/s
dt

the displacement is ( AF )2  (FD) 2

= 7 2m
It can be seen that velocity of approach is always
constant. 2. Max. frictional force
100 fmax = N
 P reaches O after = = 20 sec. = (mg + F sin53°)
5
4
= 0.2 (20 × 10 + 30 × )
5. a = (kx – f)/m 5
f × R = mR2 / 2 × a/R N

6. by energy conservation
Fcos53°
53°
1 1 mR 2 v 2
1
N
 20  0.3  0.3    2
 mv 2 mg F
2 2 2 R 2
Fsin53°
mv2 = 1.2 J = 44.8 N
mv 2 As applied horizontal force is Fcos53°
Rotational K.E. = = 0.3 J = 18N < fmax, friction force will also be 18 N.
4

DPPS FILE # 218

HAVE CONTROL ¾® HAVE PATIENCE ¾® HAVE CONFIDENCE Þ 100% SUCCESS


3. Potential gradient = 1.6 v/m
E.M.F. = potential gradient × balancing length
DPP NO. - 49
1.2 = 1.6 × 
1. Four lines, perpendicular to lines of electric field
1 .2 3 and passing through A, B, C and D are drawn.
= = = 0.75 m = 75cm
1 .6 4 These are equipotential lines. As potential
decreases in the direction of electric field, therefore
g VA > VB > VD > VC
4. mg = m2 R ,  =
R
y

KQ K 4Q C A 30°
6. = (Since r >> R) x
x2 (3 r  x ) 2
E
D
 (3r – x) = 2r
x=r
7. Once the charge reaches the neutral point it will be
accelerated towards center of ring 2, will cross it, be ds 1
reatarded, come to rest and then return towards it. 2. For A, = VA =
dt 3
Thus the motion is oscillatory, but not SHM.
ds
For B, = VB = 3
dt
4 KQ q KQ q
8. Uin = + 3r VA
R 1
VB = .
3
Its energy when it reaches the center of ring 1

KQ q 4 KQ q
Ufin. = + 3r + K.E. = Uin
R
 K.E. is positive

3.
DPP NO. - 48

If pot. drop between A and B is also 2V, then no


1 currrent will pass through the gelvanomter.

2 3 4  12 
+ Pot. drop across R =  R = 2
1. R  5
 
2Î 0 2Î 0 12 R = 2R + 10
R=1

Electric field due to both the plates will be


4. Let the angular speed of disc when the balls reach
cancelled out for all the points. So the net electric
the end be .
field at the points will be governed only by the
From conservation of angular momentum
sphere. Farther the point from the sphere, lesser
the magnitude of electric field. 1 1 m 2 m 2
mR2 0= mR2  + R + R 
Therefore E3 = E4 > E2 > E1 2 2 2 2

0
2. Since the block slides down the incline with uniform or  =
3
velocity, net force on it must be zero. Hence mg sin
5. In second case due to psuedo force acting on the
must balance the frictional force ‘f’ on the block.
block its acceleration will be more as compared to
Therefore f = mg sin = 5 10 ½ = 25 N.
the first case.
Hence t1 > t2
Ans. t1 > t2

DPPS FILE # 219

HAVE CONTROL ¾® HAVE PATIENCE ¾® HAVE CONFIDENCE Þ 100% SUCCESS


6. V = E + ir (during charging) 5. From given data
= 14 V. 
VM  5 2 î  5 2 ĵ
7. P = I2 r (Due to internal resistance)
velocity of man
= 502 × 4 × 10–2 = 100 W
Velocity of wind
8. Rate of charging = E.I. 
VW  5 2 î
= 12 V. 50 A = 600 W
The flag will flutter in the direction in which wind is
blowing with respect to the man holding the flag.
DPP NO. - 50   
 VWM = VW – VM
1. Initially :–
Vv + VA = 6 ..(1) 
VWM = ( 5 2 î ) – (5 2 î  5 2 ĵ )
; VV & VA being the potential across voltmeter &
ammeter respectively 
VWM = – 5 2 ĵ = 5 2 ( ĵ )
after closing S1 .
This implies direction of wind with respect to man in
Vv
+ 2 VA = 6 ...(2) south.
2
Flag will flutter in south direction. Ans.
Solving (1) & (2)
Vv = 4, VA = 2 .
after closing S2 : –
Vv = 0 1 C X

VA = 6
So that value after closing S2 is 3/2 times the value 20V 0V
after closing S1 . 6. A
P
B

10 V 5V
2. [ Ans: V1 = , V2 = ,]
3 3
Let reference potential of B be zero. No current
shall flow through galvanometer.
 RT
3. The speed of sound in air is v = If VC – Vp = 16 volts.
M
Now Vp = 2 volts.

of H2 is least, hence speed of sound in H2 shall be  Vc should be 18 volts.
M
maximum. VA  VC VC  VB
Now =
1 X
Solving X = 9 .
4.
1 C X

20V 0V
A B
Decrease in PE = Gain in rotational K.E. P

L
Mg + MgL = R3  R1  R 2
2
7. Balance point is independent of r. It can be seen
2
1 4 ML for balance point at P, VC – VP = E in absence of
. ML2. 2 = ML2
2 3 3 cell, jockey and galvanometer.

3 2 4ML2 8. For balance point at P.


 MgL = .ML2 . 2 =
2 3 3 VC – VP = E = 12
 VC = 18 , VP should be 6 volts.
9g 9g
 = 2  = Therefore
4L 4L
9g VA  VP V 0
 P or  = 70 cm.
[ Ans.: = ] 100  
4 

DPPS FILE # 220

HAVE CONTROL ¾® HAVE PATIENCE ¾® HAVE CONFIDENCE Þ 100% SUCCESS


DPP NO. - 51 V
Vmax

E 2E
1. E – ir1 = 0  i = and i = r  r  R 4.
r1 1 2

x=x0 x
Therefore R = r1 – r2
=2 x = x0 is the point where potential is maximum. So, if
the impulse is sufficient enough and point charge
crosses the maximum PE barrier than point charge
will move to infinity otherwise it will perform oscilla-
tory motion and for very small impulse the motion
50V may be SHM.
qi=200/3C
q f=100C 2F
4F Sol.(69-72)
q1=100/3C
1 Resitance of wire AB is -
2. S
2  1 
2F q2=50/3C dx 
 0   24  
R 
qi=200/3C q f=50C 4F
RAB =  
 2  A
=
2
= 12 

 A 

0 

15
Current in wire AB is  = = 1A
12  3
when switch is open, null point at C (AC = x)
Initial and final charges are marked on 4f
and 2f capacitors as shown. 2
  0 x   x   0 x 2 24
RAC =     = = 3 = 8
Hence charge passing through segment 1 and 2 are  2  A 2A 2
EMF E = 1 × 8 = 8 V
100 50 when switch closed null point at D (AD = x)
q1  C q2  C
3 3 1
  0 x   x   0 x 2 24
RAD =    = = 2 = 6
 charge through switch = q1 + q2 = 50 C.  2  A 2A 2
V battery = 6 × 1
V cSVjh = 6 × 1
8
8– r =6
50V r3
qi=200/3C r=1
q f=100C 2F
4F q1=100/3C
1
DPP NO. - 52
S
2 q2=50/3C
2F
2. (C) For pipe A, second resonant frequency is third
qi=200/3C q f=50C 4F
3V
harmonic thus f = 4L
A
For pipe B, second resonant frequency is second
2V
3. The equation of pressure variation due to sound is harmonic thus f = 2L
B

ds d 3V 2V
p=–B =–B [s sin2 (t – kx)]
dx dx 0 Equating, 4L A = 2L B
= B ks0 sin (2t – 2kx)
4 4
 LB = LA = .(1.5) = 2m.
3 3

DPPS FILE # 221

HAVE CONTROL ¾® HAVE PATIENCE ¾® HAVE CONFIDENCE Þ 100% SUCCESS


6. Torque equation
 Hinge = Hinge 
5. [ Ans: VAB =
1  3    = 10V ]
2

 m( 4 )2 
2
The distribution of charges is shown in fig. In mg  =  12  m  

closed loop (1)  

q (q – q1) 3g
– – 0 ...(i)
C1 C2 =
7

1q 1 q 1 q–q 3g
In closed loop (2) – C –C  C =0 Tangential acceleration =  =
1 2 2 7

 2C1  C2  Radial acceleration = 2  = 0


or q =  C  q
 1
 1  3g
Ans.
7

 3g 
7. mg – N1 = m  
 7 

4mg
N1 =
7
N2 = 0

8. Energy conservation
q1 q q 1 7
From Eq. (i), – –  1 =0 mg  = . m2 2
C1 C2 C2 2 3

q1  C1  C2  6g
or  q 
 CC 
 =
C2  1 2  7

q1  2C  C2   C  C2 
or    1  q1  1
  CC 
 DPP NO. - 53
C2  C1   1 2 
1. Moment of inertia of semicircular portions about x
 C2C12 and y axes are same. But moment of inertia of
 q1 =
C12  3C1C2  C22 straight portions about x-axis is zero.
x
– q1 q  x < y or 1
  A – B   1 y
C2 C2

2. As voltage applied across capacitor is same i.e.


 C12
= 10V in both case. Therefore in both case
C12  3C1C2  C22
10
Ed = 10  E = , as d is constant . Therefore
d

electric field remians the same as 10 V/m
C C2
= 1  3 2  22
C1 C1
3. after collision


= 1  3  2 = 10V

 C 
 2    2 
 C 
 1 
By momentum conservation in horizontal direction

DPPS FILE # 222

HAVE CONTROL ¾® HAVE PATIENCE ¾® HAVE CONFIDENCE Þ 100% SUCCESS


V = V1 + V2 .............(i)
p  2
and  = R  r . R. 3  ............... (2)
 
V2 – V1 1
e=  .............(ii) From (1) & (2) R = r. Ans.
V 2
Alternate Solution :
3V If in second case both S1 & S2 are closed,
By (i) and (ii) V2 =
4
p2r  
So impulse on B = . = p
.
r  2r  2 3
 3V 
=m   p 2rx 2
 4  = . .
2rx 2r  x 3
r
and loss in K.E. 2r  x
3
= mV2 p 4 p x
16  = .
3 2r  x  2x

4. Ans. 3000
Sol.(57 to 59)
Time period is minimum for the satellites with
minimum radius of the orbit i.e. equal to the radius
of the planet. Therefore.

GMm mv 2

R2 R
Final potential of spheres will be same
GM x Ky K ( Q  x  y )
V= So, K = =
R R 2R 3R
y = 2x and 3x = –Q – x – y
2R  6x = –Q
T min =
GM
Q Q
R x=  y= 
6 3

2 R R Q
= Charge on sphere of radius 3R is 
GM 2
Change in potential energy of sphere of radius ‘R’ is
4
using M = p R3. KQ 2 K ( Q / 6)2
3 U = 
2R 2R
3
T min. = 35KQ 2
G U = .
72R
Using values T min = 3000 s
DPP NO. - 54
5. When S1 is closed
1. At highest point
p p
i= = 2m m
2r  r 3r mu = v – 2u
3 3

p p 2u 2
 = .r= ............. (1) mu + = v
3r 3 3 3

when S2 is closed (Let resistance of w2 be R) 5u


 v=
2
p
i= 5R R 7R
Rr
total horizontal distance = + =
2 2 2 4

DPPS FILE # 223

HAVE CONTROL ¾® HAVE PATIENCE ¾® HAVE CONFIDENCE Þ 100% SUCCESS


V V 4
8. Ex = – =–
v x 1

2. (i) Vcm 4–8


Ez = – =4 3
 smooth 1/ 3

|E | = E 2x  E 2z = 64 = 8 N/kg Ans. (A)


Vcm + R = V
Vcm = V – R

 depends on value of friction between plank &


cylinder, hence Vcm is undetermined.
2v V 2V V
(ii)  = = (iii)  = =
2R R 2R R
Vcm =0
3V – V 2V 2V DPP NO. - 55
(iv) A/C = = =
R R R

3.  wheat stone bridge is in balanced condition KQ 2 KQ 2 KQ


1. Ei =   .Q
100 x 2a 4a 2a
100 100 x
So =
1 2 KQ 2 KQ 2 5 KQ 2
=  =
a 4a 4a
100
100 K (2Q)2 KQ 2
Ef = =
x 4a a

KQ 2
Ei – Ef = H =
1 2 4a
2. In potentiometer wire potential difference is directly
1
proportional to length
 2 = 2
 x = 100 

5. As gravitational force provides centripetal force

mv 2 GMm
=
r r3
GM
i.e., v2 =
r2

2r r2
So that T = = 2r Let potential drop unit length a potentiometer wire be
v GM
2 4
K.
 T  r
For zero deflection the current will flow indepen-
Ans. 4
dently in two circles
R = K × 10 .... (1)
1
6. VA + × 1× (2)2 = – 3 R + X = K × 30 .... (2)
2
(2) – (1)
VA = – 5 Ans. (C)
 X = k × 20 .... (3)

GM R 1
7. – Ans. (C) (1)/(2) = 
R X 2

DPPS FILE # 224

HAVE CONTROL ¾® HAVE PATIENCE ¾® HAVE CONFIDENCE Þ 100% SUCCESS


3. Let  be the charge density of conducting plate and
v
V be the volume of either dielectric 6. (A) 5 m/s
A
1 from linear momentum conservation
E12  V

 k1 0
U1 2  45
 U2 =  1 2 M AV = m b 5  v = = 0.5 m/s Ans.
 k 2 0 E2  V 40
2 
7. (C)
m A 0.5 + m b 5 = (MA + m b) V1
2
  
  40  0.5  4  5 40 10
k1  k 1 0  k2 V1 = = = m/s Ans.
= = 44 44 11
k2  
2 k1

  8. (C)
k
 2 0  after throught the ball velocity of man A is 0.5 m/s
4. 2 For man B
PE = –4 MJ 4 × 5 = 40 v2 – 4 × 5  v2 = 1 m/s
TE = –2MJ velocity B is 1 m/s after through the ball
The additional energy required to make the satellite after through the ball second time, velocity of man
escape = +2MJ. A is
4 × 5 + 40 × 0.5 = 40 × v3 – 4 × 5
5. Applying conservation of mech. energy. v3 = 1.5 m/s
similarly for man B v4 = 2 m/s
after 5 round trip and man A hold the ball velocity
of man B is 5 m/s
velocity of man A
4.5 × 40 + 4 × 5 = (40 + 4)v5

50
v5 = m/s Ans.
11
9. (A)
When man through the ball 6 times it velocity is
greater than 5 m/s and velocity of B is 5 m/s
therefor maximum number of times man A can
through the ball is 6 .

10. Fext = 0 , Centre of mass of system cannot move


decrease in P.E. = increase in rotational K.E.
Initial position of centre of mass from A.
 3R  1 44 40
 (2m) g 2   =  system 2  
 4  2
(A+ball) d B
1
3 mgR = [ + mass ] 2 40 d 10
2 disc. X cm = = d
44  40 21
3 mgR

1  mR 2 mR 2 25mR 2  2
DPP NO. - 56
= 2  4  16  16  
  1. Potential drop.  V = i (R + RA)

3g 1  30  2 16 g V
    = = R + RA = Rmeasured.
R 2  16  5 R i

5 
Vparticle =  R    5gR
4 

DPPS FILE # 225

HAVE CONTROL ¾® HAVE PATIENCE ¾® HAVE CONFIDENCE Þ 100% SUCCESS


4. When charge on plate is constant electric field 3. Switches open :

Q
remains constant E = R1
2A 0
A
In case when potential difference is constant E =

V R
R3
d
R2 E
Electric field increases when 'd' decreases and
hence chances of breakdown increases.

5. C = k 0A /d
Af = 4A i E
A = R  R  R
df = 2 di 1 2 3
Because, ALL linear dimensions are doubled so Switches closed :
capacitance become doubled. There will be no current through R3.
Current through E and R2
6. x = 3 sin 100 t + 8 cos2 50 t
E
8 [1  cos 100 t]  =
R R1
= 3 sin 100 t + R2 
2 R  R1
x = 4 + 3 sin 100 t + 4 cos 100 t Current through the ammeter
 4 R  RE
(x – 4) = 5 sin (100t + ) tan     A = R  R = (R  R ) R  R R
 3
1 1 2 1
Amplitude = 5 units
Maximum displacement = 9 units. E
= R 2R 1
R1  R 2 
7. C = k 0A /d R
formula suggest that it depends on area, separation As A =  A
and surrounding medium.
R1 R 2 100  300
R= R3 = = 600 
9. Most Appropriate capacitor is a capacitors of high 50
capacitance & high dielectric strength. By dielectric
strength
4. 4 times
C>B>A>D
k 0 A A1 = A3 = 8 (area)
By capacitance C =
d A2 = A4 = 9
C=D>B>A Position of the particle at any time t is given by
So C is best. t

X = X0 +  Vdt
0
X0 = Initial position
DPP NO. - 57

T 1 
2.  100 =  100 is not valid as  is not
T 2 
small.

 2 T2  T1
T1  2 T2  2 % change = ×
g g T1

100 = ( 2 – 1)  100 = 41.04.

DPPS FILE # 226

HAVE CONTROL ¾® HAVE PATIENCE ¾® HAVE CONFIDENCE Þ 100% SUCCESS


t
4.  and Y are properties of material.
These coefficients are independent of geometry of
 Vdt = Area under the curve
0 body.
5. Relative to liquid, the velocity of sphere is 2v0
Now at t = 0 X = X0 = – 1
upwards.
at t = 2 X = X0 + A1 = – 1 + 8 = 7
at t = 5 X = X0 + A1 – A2 = – 1 + 8 – 9  viscous force on sphere
=–2 = 6r 2v0 downward
t=7 X = X0 + A1 – A2 + A3 = – 1 + 8 – 9 + 8 = 12r v0 downward
=6
t = 10 X = X0 + A1 – A2 + A3 – A4 = – 1 + 8 – 9 + 8 –
7=3
As during 10 seconds four times the position of the
particle changed in sign. Particles passes 4 times 6. 
the origin

5. Once the switch is closed, the capacitor is charged


through resistance R1 by the battery's e.m.f. Time
constant is R1 C.

6. Using Vc = E (1 – e–t/RC) 
 Vc = 12 (1 – e–2)
= 10.4 V

7. At any moment in the circuit –


0 A / 2  A/2 4 0 A
Vc + VR1 = 12 V C1 = , C2 = 0 = C
d d/ 2 d 5d

 VR1 = 12 V – 10.4 V k 2

= 1.6 V
13  0 A 13 0 A
= C1 + C2 = Ans.
10 d 10 d
8. If loop law is applied to the left hand loop in clockwise
direction
E – VR2 = 0 1
7. The current through the galvanometer is ~
1000
VR2 = E = 12 V
of total current, the S << G.
i.e. VR2
does not change during the charging process.
8. Potential difference across galvanometer =
Potential difference across S.
DPP NO. - 58  ig . G = ( – ig) . S
 10 × 10–3  10 = (1 – 10 × 10–3)  S
1. By symmetry
AB = BC & AD = DC 10 1 10
 RS = 2 = 
 No current in BO and OD 1  10 99
 TB = TO = TD
DPP NO. - 59
2. In (A) xf – xi
0 – x = – x = – ve 3. (A) Moment of inertia of the rod w.r.t. the
So average velocity is – ve. axis through centre of the disc is : (by parallel axis
theorum).
3. Speed of block is maximum at mean position. At
mean position upper spring is extended and lower mL2
=  mR 2
spring is compressed. 12

DPPS FILE # 227

HAVE CONTROL ¾® HAVE PATIENCE ¾® HAVE CONFIDENCE Þ 100% SUCCESS


DPP NO. - 60

2. The velocity of the body a time t is given by

dx d  t2  t
   
dt dt  4  2

 At t = 0 ,  = u = 0 and t = 2 s,  = 1ms-1, Now,,


& K.E. of rod w.r.t. disc work done = increase in KE

1 2 1 1 1
=
2
  m 2  mu 2  m 2  0
2 2 2

1 2
 2 L2 
= 2 m  R   Ans.
 12 
1 1
 m 2   6  (1) 2
2 2 = 3J,
5. Since, Heat energy radiated per sec = AeT 4
where, e is emissivity of the surface which depends
upon its nature and A is its area. Hence the correct choice is (d).
T is its own tem per atur e (independent of
surrounding temperature) 3. From conservation of energy, the kinetic energy of
Hence, (A) and (C) are correct. ball at lowest portion is (vc = speed of centre of
ball)
8. Let VA = 0 volts. 1 1 2 2
 Net current entering node C = 0 mv c2 +  mv c = mgR
2 2 5
0  Vc 2  Vc 1  Vc
  0 7

2 6 .5 2 or mv 2c = mgR
10
1 Since net tangential force on sphere at lowest point
 VC = volt. = p.d across wire AC.
6 is zero, net force on sphere at lowest position is
VC  VB VB  2 VC  2 1 mv c2 10
Also = =  =  mg upwards.
5 1 .5 6 .5 3 R 7

5
 VC – VB = = p.d. across wire BC
3 R . RV
6. RA = R  R < R
 VC > VB > VA V

Hence potential gradient across BC = 7. RB = R + RG > R

2V -2v 1.5v
8. % error in case A.
VC 5  RV
A VB RA  R 
OV C × 100 =  R  R  1 × 100
R  V 
1V 1v
R
= R  R × 100  - 1%
V
5 / 3 10
 V/m
1/ 2 3 % error in case B
also potential gradient across RB  R R
× 100 = G × 100  10%
R R
1/ 6 5
AC  V/m Hence percentage error in circuit B is more than
1/ 5 6
that in A.

DPPS FILE # 228

HAVE CONTROL ¾® HAVE PATIENCE ¾® HAVE CONFIDENCE Þ 100% SUCCESS


5. By W net = K.E = 0
DPP NO. - 61
1
 k(x02 – x2) = mgx
2. 11 = 22 2
Since, men move towards middle of turn table 2
1 1
decreases hence 2 increases.  × 200(22 – x2) = × 60 × 10x
2 2
1 1  x = 1m
 K =   2 – 222
2 1 1 2 Also at this moment fmax > kx
So, block will not move so total distance travelled =
  2 2 2  2 + 1 = 3m.
1 2 1  . 2
 2 
 1
=  
2 1 1  1 1   1 
6. In steady state, before the switch S is closed,
 2  potential difference across capacitor is 40 volts. Just
1 2
= 11 1   <0 after switch S is closed, charge and hence potential
2  1 
difference across the capacitor does not change
So kinetic energy increases. appreciably. So, the potential difference across R2
is 40–10 = 30 volt. The curent through R2 is 3
ampere.
3. (A,B) For steady state
7. The current through resistors when the capacitor
 dQ   dQ  is in steady state with switch S closed.
  =  
 dt  in  dt  out 40  10
I = R  R  1amp . Therefore potential difference
(V) (i55) = 45(T – 20) 1 2

(500) (4.5) = 45(T – 20) across R2 is 10 × 1 = 10 volts. Hence the potential


T55 = 70ºC. difference across the capacitor is 10 + I R2= 20
volts. So, the charge on capacitor q = CV = 200C.
v 500
(C,D) Resistance at 20ºC is R = =
i 5
8. At the given instant, p.d across capacitor is 20
R20 = 100  Volts. Hence the current through R1 at the required
v 500 40  20
Resistance at 70ºC is R = = ~ 111
11  instant of time is I =  1amp
i 4 .5 – R1
Rf = R0(1 + T)
111 = 100(1 + (50))
DPP NO. - 62
0.11
= ~ 2.2 × 10–3 /ºC.
50 – dT
1. Heat current : i = – k A
4. For painter ; dx
R + T – mg = ma idx = – kA dT
R + T = m(g + a) ............(1)
 T2
For the system ; i dx = – A 
2T – (m + M)g = (m + M)a

0
 T dT
T1
2T = (m + M) (g + a) ..............(2)
where ; m = 100 kg
(T22  T12 )
M = 50 kg  i  = – A 
2
a = 5 m/sec 2

150  15 A  (T12  T22 )


 T= = 1125 N i =
2 2

and ; R = 375 N

DPPS FILE # 229

HAVE CONTROL ¾® HAVE PATIENCE ¾® HAVE CONFIDENCE Þ 100% SUCCESS


8. By mechanical energy conservation for m, just after
2 2
2. i =  VAB = × 10 collision
10  R 10  R
1
mv2 = mgL sin
2 10 2
x =   x max = 0.2 V/m.
10  R 10

m
L
  2v  r
3. B 0
4 r 3
M CM
=
3m L 0

 qv sin 
B 0
4 r 2
v2 24 2  5
 L=  = 36 meter..
2  100  sin 30 2g sin  20  4
= 10–7 ×
(2)2 Alternate :
since there is no energy loss, center of mass of m
1 and M rises to the same initial position.
2  100 
= 10–7 × 2 3mL0 = m(L – L0)
22  4mL0 = mL
 L = 4L0 = 36 meter.
= 25 × 10–7 T
= 2.5 × 10–6 T
= 2.5 T DPP NO. - 63
1. (B) Initially effective resistance = 2R. In parallel
R
effective resistance = . It has reduced by a factor
2
5. of 1/4 so rate of heat transfer would be increased
by a factor of 4, keeping other parameters same.

V0 = 5 × 2t = 10 t

1 1 4.
S = 1500 = V0 .3 t = 10 t.3t
2 2
 t = 10 sec.
 total time = 3t = 30 sec. F1L F2L L
F1  F2 – F1  F2 = 4

6. J us t bef ore c ollis ion veloc ity of M and F1 5


 F = K
m= 2gL 0 sin  = 12 m/s 4 3
[ Ans.: 3: 5 ]
Since collision is elastic, let velocity of m just after
5. (A)
collision is v then by relative velocity of separation
Distance travelled = 24t0 + 21 t0
= relative velocity of approach
v = 12 + 12 = 24 m/s Ans.

7. By momentum conservation during collision of m


and M.
12 M – 12 m = 24 m
m:M=1:3

DPPS FILE # 230

HAVE CONTROL ¾® HAVE PATIENCE ¾® HAVE CONFIDENCE Þ 100% SUCCESS


= 45 t0 2. (B)
 45t0 = 180 m Point A shall record zero magnetic field (due to -
 t0 = 4 seconds particle) when the -particle is at position P and Q
 Distance translted by A is as shown in figure. The time taken by -particle to go
24t0 = 24 × 4 = 96 m from P to Q is

Sol(57,58,59)
(57). Initial velocity of com
20 m  0
u= m  m = 10 m/s 
acceleration of com
= g = 10 m/s2 
initial height = 10 m
1 2 1 2 2
S = ut + at t= or  =
2 3  3t

1
10 = – 10t + (10) t2 20  10
2 3. Angular velocity w = = 20 rad/sec.
0 .5
5t2 – 10 t – 10 = 0
t2 – 2t – 2 = 0

2 48
t=
2
t=1+ 3 4. Potential difference across wire AB = 5 V
u 2  p.d. across 40 cm of this wire
(58). Hmax. = 10 + = 10 + 5 = 15 m 5
2g = × 40 = 2 volt.
100
 Potential difference across 20 cm of wire
2  20 2  20
(59). t = – = 4 – 2 = 2sec. CD = 2 volt.
a g
2
 p.d. across wire CD = × 80 = 8 volt.
20
DPP NO. - 64
p.d. across 2  resistor = 2 × 2 = 4 volt
 Emf of the cell = 12 volt.
dQ KAT T 10
1.  = = J /sec.
dt 2 2 120
2 2 1
KA 5.  v ds =  ds dt ds = 2t = 8 × 4 + 2
× 10 × 2

dQ
 T t = 21 s
New rate =
dt  t – 20 = 1 s Ans.
2 KA
Sol. (1 to 3)
40 mA × 0.8 = mA × 0.2 + mB × 1.0
= J /sec. ;
120 mA × 0.6 mB × 1.0 mB = 0.6 mA
1 – 0 .2
e= =1 = 1.5
0 .8
d = 6 × 0.5 – 6 × 0 = 3N – 5
= 10 × {0.8 – 0.5} = 10 × 0.3
= 3 NS

20 1 1
So time taken is t = U = × 10 × (0.8)2 – × 10 × (0.5)2
40
× 120 sec. 2 2
= 60 sec. = 5 × 0.64 8 × 0.25 = 3.2 – 2.0 = 1.2 J

DPPS FILE # 231

HAVE CONTROL ¾® HAVE PATIENCE ¾® HAVE CONFIDENCE Þ 100% SUCCESS


2
DPP NO. - 65 ma 2  a  7ma2
5.  = + m  
 = = 7.
12  2 12

2
1 1 1 2  2v 
6. mgh = mv2 + mv2 + . mv2  
2 2 2 5  r 

1 8 1 18 9 mv 2
= mv2 [1 + 1 + ] = mv2 =
2 5 2 5 5
1.
5
 v= gh
9

2
1 1 2  2v  13
7. KE of the ball = mv2 + mv2   =
2 2 5  r  18
M
Mx + (x + L) = 0 mgh
3

4M ML L 2h
x=– x=– 8. X = 2vt = 2v
3 3 4 g

5 2h 2 10
= 2. 9 gh g = h
  
0 v  r 1 qr
3
B = 4 q r 3
 
2. and E = 4 0 r 3

  DPP NO. - 66
   v E
 B = 0  0 ( v  E ) = c 2
1. Initially the centre of mass is at
L
distance from the vertical rod.
50 1 2GM 4
3. v = Ve =
100 2 R 1
 As, x  m ( 2 )  m (0)  L 
 
Applying energy conservation cm
 mm 4
 
GMm 1 GMm
   mv2 = 
R 2 (R  h)

2 GM 2 GM
v2 = 
R Rh

1 2GM 1 1 
 . = 2GM   
4 R  R R h

1 h
  centre of mass does not move in x-direction as
4R R(R  h)
Fx = 0.
 R + h = 4h After they lie on the floor, the pin joint should be at
 h = R/3 L/4 distance from the origin shown inorder to keep
the centre of mass at rest.
4. Keq. is same in all three cases. All other parameter
being same, rate of energy conduction is same in all L
 Finally x-displacement of the pin is and
three cases. 4
Simlarly temperature difference across any material y-displacement of the pin is obviously L.
in any wall is also same.
L2 17 L
Hence net displacement = L2  
16 4

DPPS FILE # 232

HAVE CONTROL ¾® HAVE PATIENCE ¾® HAVE CONFIDENCE Þ 100% SUCCESS


6, 7 & 8.
dT dT H
2. H = – kA  = Let  be the angular acceleration of rod and a be
dx dx kA
acceleration of block just after its release.
dT  mg – T = ma ..... (1)
Now as k increases, becomes less (–)ve
dx
 m2
So slope becomes less (–)ve T – mg =  .... (2)
2 3
So curve will be and a =  .... (3)

3. Since
 
0 v  r   Solving we get
 q
B = 4 r 3 , v  r must be same
5 mg 3g

where v = velocity of charge with respect to T= and  =
8 8
observer
Now from free body diagram of rod, let R be the
Let A and B are the observers
      reaction by hinge on rod
then (v C  v A )  r = (v C  vB )  r
1
   R + T – mg = m acm = m 
or ( v A  v B )  r  0 2
   Solving we get
or ( v A  v B ) || r
4. 11 = 11 (Angular momentum is conserved) 9 mg
R=
As 2 decreases. 2 increases. 16

2
Thus T = i.e. T decreases.

Therefore the earth is completing each circle around
its own axis in lesser time.

1 2 DPP NO. - 67
K.E. = 
2 
Therefore K.E. of rotation increases. 1. Moment of inertia is more when mass is farther from
Duration of the year is dependent upon time taken the axis. In case of axis BC, mass distribution is
to complete one revolution around the sun. closest to it and in case of axis AB mass distribution
is farthest .Hence
5. Using  axis theorem A

5
3 cm
x y
B C
IBC< IAC< IAB
 I P > IB > IH I C = ICM + my2
1 2 2
x = y = IB – mx + my
2x = 1.6 = IB1 + m (y2–x2) = IP + IB + m (y2 – x2)
x = .8 Ma2 > IP + IB
AB = x + M(2a)2 > IP
= 4.8 Ma2 Here IB1 is moment of inertia of the plate about an
Ans.: 4.8 Ma2 axis perpendicular to it and passing through B.
 IC > IP > IB > IH

DPPS FILE # 233

HAVE CONTROL ¾® HAVE PATIENCE ¾® HAVE CONFIDENCE Þ 100% SUCCESS


2. (C) Taking the origin at the centre of the plank.
Qv
=
2 R
40 kg 60 kg The magnetic field at point P is
A B
µ0 2R 2 µ0 QvR
B= =
smooth
4 2
(x  R ) 2 3/2 4  (2R 2 )3 / 2
40 kg

60 cm E 1 c2
 = µ  v =
B 0 0 v

5. The orbital velocity,

x GM
v0 =
r
Its velocity is increased by 2 times, new velocity
m 1x 1 + m 2 x 2 + m 3x 3 = 0
(  x CM = 0) GM 2GM
v= 2  = escape velocity
(Assuming the centres of the two men are exactly r r
at the axis shown.) The path is parabolic in case of escape velocity.
60(0) + 40(60) + 40 (–x) = 0 , x is the displacement
of the block.
8. L =   dt =  x Fdt = x P ( bec ause x is
 x = 60 cm essentially constant during the quick blow)
i.e. A & B meet at the right end of the plank. since, the rod starts at rest, the final values
therefore satisfy L = xP.
3. The slope of temperature variation is more in inner 1  12 x
 m2  = xmv   2 .... (1)
2 v
dQ KA 
= .T 
dt  Another expression for is obtained from the given
v
 dQ information that rod makes one revolution by the
T = . time centre reaches the dot.
KA dt
t = 2 and vt = d
1  2
Slope    .... (2)
K v d
Larger the conductivity, smaller is the slope. 12x 2  2
from equation 1 and 2 :  x=
2 d 6d
4. Let Q be the charge on the ring. The electric field at 
point P is x cannot be larger than
2
 2  
  d
6d 2 3

DPP NO. - 68

 D  25 
1. MP =  1   =  1   =6
 f   5 
1 Qx
 E = 4  2 2 3/2 2. Heat radiated (at temp same temp)  A
0 (x  R )
 Q  4R2 and Q'  (4R2 + 2 × R2)
1 QR
= 4  Q' 6R 2
2 3/2  = 1.5
0 (2R ) 
Q 4R 2
The rotating charged (Q) ring is equivalent to Here R2 is extra surface area of plane surface of one
a ring in which current  flows, such that of the hemisphere.

DPPS FILE # 234

HAVE CONTROL ¾® HAVE PATIENCE ¾® HAVE CONFIDENCE Þ 100% SUCCESS


7. If m is pole strength , then
M
m =m 
l
3. When the wire is bent into a semicircular arc, the
separation between the two poles changes from l to
2l, where new magnetic moment of the steel wire,
M 2l 2M
M'  m  2r   
  l  
Megnetic moment M = r2 i î &  B = 3 î  4 ˆj
8. (A) Real image of a real object is formed by concave
  mirror and convex lens.
  = M × B = r2 (3 î – 4 î )

(B) Virtual image of a real object is formed by all four.
 (C) Real image of a virtual object may be formed by
 will be along the direction shown . all four.
Hence , the point about which the loop will be lift (D) Virtual image of a virtual object may be formed by
up will be : (3, 4) convex mirror and concave lens.
(A) p,r (B) p,q,r,s (C) p,q,r,s (D) q,s

9. Let E1 < E2 and a current i flows through the circuit.


Then the potential difference across cell of emf E1 is
E1 + ir1 which is positive, hence potential difference
4.
across this cell cannot be zero. Hence statement 1
is correct.
For current in the circuit to be zero, emf of both the
( velocity )2
Radius of Curvature = cells should be equal. But E1  E2. Hence statement
Normal Acceleration
2 is correct but it is not a correct explanation of state-
( 2v ) 2 ment 1.
= = 4R
v2 / R

5. By linear momentum conservation in horizontal di- E1, r1 E2, r2


rection = for (bob + string + cart)
mV0 = (m + m)v
V0
v= DPP NO. - 69
2
By mechanical energy conservation for
(bob + string + cart + earth) 3. Equal area means equal power output. A3 area
pertains to highest wavelength range, thus photons
1 1 with minimum range of frequency. Thus maximum
mV02 + 0 + 0 = (2m)v2 + mgh + 0
2 2 number of photons are required from this segment
2
to keep the power same.
1 1 V
mV02 – (2m) 0 = mgh
2 2 4 4. Work done by kinetic friction may be positive when it
Solving it, acts along motion of the body.
Friction on rigid body rolling on inclined plane is along
V02 upward because tendency of slipping is downwards.
h= .
4g
6. When two drops of radius r each combine to form a 5. The torque of system = Torque on loop
big drop, the radius of big drop will be given by [AFGH + BCPE + ABEF]

4 4 2 4  3 = SB ( î ) + SB( î )+ SB k̂ ( I = current,


R 3 = r  r
3 3 3 S = area of loop, B = magnetic field.
or R3 = 2r3 or R = 21/ 3 r Now =  S B k̂
2 2 1 = 1 × 1 × 2 k̂ = 2 k̂ units
VR  R 
   = 23  43
Vr  r  [Ans: 2 K̂ ]
 VR= 5 × 41/3 cm/s

DPPS FILE # 235

HAVE CONTROL ¾® HAVE PATIENCE ¾® HAVE CONFIDENCE Þ 100% SUCCESS


6. The work done to rotate a bar magnet from its initial
position  = 1 to the final position  = 2 is given by
W = M B (cos 1 – cos2), A B
(i) Here 1 = 0° and 2 = 180° R1 R2
 W = M B (cos 1 – cos180°) = M B = [1–(–1)] = 2 R3 E
E
MB
(ii) Here 1 = 0° and 2 = 90°
Fig-1
 W = M B (cos0° – cos 90°) = M B = [1– 0)]
= MB

7. If velocity of m2 is zero then


by momentum conservation
m1v = m2 v

m2 v
v = m
1

Now kinetic energy of m1


Therefore statement 1 is true. Statement 2 is obvi-
2
1 1  m2  2 ously false.
= m1 v2 = m1   v
2 2  m1 

 m2  m  m
DPP NO. - 70
1   m v2 =  2  1 m v2 = 2 × initial
= m1
2  m1  2
 m1  2 2
1. In normal adjustment
Kinetic energy
f0
Kinetic energy of m1 > initial mechanical energy of m= f
system m1 e

Hence proved
100
so 50 = f
e
 1 
8. C = sin–1   = 30°  fe = 2 cm
 2 / 1

for i = 37 TR ( eyepiece is concave lens)


so,  =  – 2 (37°) = 104° and L = f0 – fe = 100 – 2 = 98 cm

 2.  m T = const.
i = 25, Refraction < –C
2 n  m + nT = C
i = 45°, TR
d m dT d m dT
 0   = 
 m T m T
so,  =  – 2   = 90°
4
d m 1
By applying snells law for prism : Now  =–1% =  (–ve sign indicates
m 100
i = 90
decrease)
r1 = 30
dT = 1 (given)
r2 = 30
 T = 100 K.
e = 45
 = 90 + 45 – 60 = 75°
3. Emissive power =  T 4
= 6 × 10–8 × 1004 W/m 2
9. The points A and B are at same potential, then under
given conditions points A and B on the circuit can be
connected by a conducting wire. Hence the circuit
can be redrawn as shown in figure 2.

DPPS FILE # 236

HAVE CONTROL ¾® HAVE PATIENCE ¾® HAVE CONFIDENCE Þ 100% SUCCESS


7. w.r.t. the wedge

4.

to reach  = 270º, it has to cross the potential


energy barrier at  = 180º and to cross  = 180º
angular velocity at  = 180º should be 0+
k i + Ui = k f + Uy

13 
 MR 2  2 + (–Mi AB cos 0º) = 0 + (–NiAB
22 
As maximum height = 125 m
cos180º)  block want by a height 20m over the wedge
=  (v sin53º)2 = 2.g.20
80  9 rad/sec.
16
v2 = 400
25
5. Equation for linear motion
mgsin – f = ma v2 = 25 × 25
for rotary motion v = 25 m/sec.
 block left the wedge with a relative velocity
25 m/sec.
Now, time of flight = 2 + 5 = 7 sec.
horizontal range w.r.t. wedge
= vx × T
= 25 cos 53 × 7
= 105 m.

a 
f. R =  .  f= 2 . 8. For slab no deviation so  = 0 for any i
R R
for slab for light from D to R
=r–i ....(i)
MR 2 3
a mgsin = ma + .a = ma nd sin i = nr sin r
2 2
2.R
 nd 
2g sin  g  r = sin–1  n sin i
a= =  r 
3 3

1 2  nd 
using S = ut + at for linear motion.  = sin–1  n sin i – i
2  r 

1 g 2 it is non-linear function and graph is


1=0+ . .t
2 3

6
t= g sec. Ans.

6. Here BH = 0.22 T ; BV = 0.38 T

2 2 After i > C T.R. will occur and graph is straight line


Now B  BH  B V
for D to R
= (0.22)2  (0.38 )2  0.1928 = 0.44 T

DPPS FILE # 237

HAVE CONTROL ¾® HAVE PATIENCE ¾® HAVE CONFIDENCE Þ 100% SUCCESS


3. Due to the motion of the loop, there will be an
induced current flowing in the circuit, resulting in
a force acting on each element of the loop equally
& radially. Therefore the net force on the loop is
zero.
Hence (D).

Similarly for light going from R to D

 nr  5. Decrease in PE =
=i– sin–1  sin i
 nd  Increase in rotation K.E
and for prism graph is drawn from   1
i = imin to i = e that is graph(s)  2mg. – mg. =  . 2
2 2 2

9. Statement-2 is wrong as in this case  2 


=
1  2m   m.   2
A B 2 
 4 4  

E
mg 1 3m 2 3m  2 2
= . .= 
2 2 4 8
E r
4g  4g g
= and v = r = =
A is at high potential and B is at low potential and 3 2 3 3
there is no current from A to B. It also justifies
Statement-1. [ Ans.: (a) V = g / 3 ,  = 4 g / 3 ]

DPP NO. - 71
1. By right hand thumb rule, the field by both the
segm ent s a re out of t he pl ane i . e. al ong
+ve z-axis. 6.

2. Let us compute the magnetic field due to any one


segment :

mv12
mg cos  + T1 =

for leaving circle T1 = 0
mv 12 = mg  cos  ...(i)
and by energy conservation
 0i
B= (cos 00  cos(180   )) 1 1
4(d sin  ) 0+ m ( 3g ) 2 = mv 12 + mg (   cos )
2 2
 0i
= 1  cos   =  0i tan  1 1
4(d sin  ) 4d 2 m (3g ) = mv 12 + mg (1 + cos)
2 2
Resultant field will be :
0i   0i 3mg mg cos 
tan = + mg + mg cos
Bnet = 2B = k = 2 2
2d 2 2d

DPPS FILE # 238

HAVE CONTROL ¾® HAVE PATIENCE ¾® HAVE CONFIDENCE Þ 100% SUCCESS


(by eqation (i)) Y

mg 3 upto
= mg cos  D
2 2 O X
a
1
cos  = a B
3 Z  parallel to 'y' axis

1 8 A
sin  = 1 = upto
9 3

v 12 g cos 
ac = = = g cos 
 
2.
at = g sin 
ac
then
at

The magnetic flux must remain constant


g cos  1/ 3 1 1 1
= = = = = B0
g sin  8 /3 8 2 2 y 2
m = B0ab = bx
1  kt
so y = 2
where x is as shown
Ans. y = 2
 x = a(1 + kt)
dx
or v = = ak Ans.
dt
Sol. (7 to 9) 3. (A) Let 'F' be the magnitude of force exerted on
Applying bernoulli's equation the rod due to the collision.
1 h Then : F = ma
P0 + × 2 × V2 = P0 + 2g × + gh
2 2  m 2
and F. = .
4 12
v= 2gh (about 'O')

1 h  a=  .... (1)
× g × t2 = 3
2 2 1 2 1 2
Using ; S = ut + at and  = 0 t + t
2 2 
h 1 2 1 2
 t= g S = ut + at  = 0 t + t
2 2 
1 2 1 2
R=v×t S= at and 6 = t
2 2 
 2h 6  3
 =  (from (1)) )
Applying continuity equation s a 
 S = 2  Ans.
6 × 2gh = 3gh × A
A = 2cm 2
mv 2
DPP NO. - 72 5. = N – mg sin
R

1. BOD = 0
BOB = 0

0
BAB =
4 a 2 [cos 45 (  î )  cos 45 k̂ ]

0
= (  î  k̂ )
8a

DPPS FILE # 239

HAVE CONTROL ¾® HAVE PATIENCE ¾® HAVE CONFIDENCE Þ 100% SUCCESS


Velocity of separation after collision = e (velocity
mv 2
N= + mg sin of approach before collision).
R
From centre of mass frame in a head-on collision,
By energy conservation,  
if u1 and v 1 be velocity of a ball before and after
1  
collision v 1   eu1 . Since, v cm = 0 from ground
mgR sin = mv 2
2 frame, ground frame and centre of mass frame carry
same meaning.
mv 2
= 2mg sin
R DPP NO. - 73
N = 3mg sin
1. For disc, from torque equation
mv 2 2
Ratio = = (constant)
RN 3 mR2
3 mg R – TR =  .... (1)
2
2
x= . By application of Newton's second law on block
3
we get,
6. Consider a block of ice having volume V and density T – mg = ma .... (2)
i.
where a = R  ..... (3)
Let the volume of ice submerged in water (of density
w) be V

M,R T
a

T mg
F=3mg

Since the ice block is in equilibrium


4g
i V solving a =
i V g = w Vg or V = ... (1) 3
w
Let V volume of ice melt in to V of water. Then 3. (A) From continuity equation, velocity at cross-
 i V = w V section (1) is more than that at cross-section (2).
i V Hence ; P1 < P2
or V = ... (2) Hence (A)
w
from (1) and (2)  V = V
Hence when V volume of ice melts it occupies V 
V volume of water.
4.

mg

N = mr2 .............(i)
Hence the level of water does not change on melting N = mg .............(ii)
of ice. From (i) & (ii),
(mr2) = mg
9. The velocity of centre of mass is always zero. At
g 10
maximum deformation during head on collision, 2 = r =
0.8  18
velocity of each sphere is equal to velocity of centre
of mass and hence zero. Therefore at maximum 5
deformation K.E. of system is also zero. = rad/s.
6

DPPS FILE # 240

HAVE CONTROL ¾® HAVE PATIENCE ¾® HAVE CONFIDENCE Þ 100% SUCCESS


5. By newton's law :
DPP NO. - 74
dv
mg  i l B = m  (1)
dt 1. When the key is at position (B) for a long time ;
the energy stored in the inductor is :
2
1 1  E  L.E.2
UB = Li 02 = .L.   = 2
2 2  R2  2R 2

q
By k v l Bl v = i R +  (2) This whole energy will be dissipated in the form of
c
heat when the inductor is connected to R1 and no
differentiate (2) w.r.t. time
source is connected.
dv di i
Bl =R
dt
+  (3)
dt c

dv
Eliminate by (1) & (3)
dt

Hence (A).
m  di i 
mg  i l B = R 
B   d t c  2. P = AeT 4
2 = 2 x 10-6 x0.9x 5.6 x 10-8 x T4
di mi
 m g B l  i B 2 l 2 = m R dt +  (4) 1014
c 4
T =
0.9 x 5.6
di T = 2110 k
i will be maximum when = 0.
dt

Use this in (4) 1 1


3. f =  metre
 m g B l c = i (B l c + m)
2 2 p 2
f = 0.5 m this is positive so lense is convex lense.
m g B c
 i max = Ans.
m B 2  2 c

6. Angle of dip can not be calculated by tangent


galvanometer. 4.

2    10 2  2  10 5
7. K = = 0.005
4  10 7  200

8. i = K tan 
using values ( = 60°)

3
i = 0.005 × 3 = Condition for pure rolling
200
V – R = V' ...........(i)

DPPS FILE # 241

HAVE CONTROL ¾® HAVE PATIENCE ¾® HAVE CONFIDENCE Þ 100% SUCCESS


Momentum conservation
DPP NO. - 75
m'V' = mV
1. Rate of heat transfer is same through all walls
mV
 V' = ...........(ii)
m' K 1.A.(10 ) K 2 A(20) K 2 A ( 40)
40 cm
= 20 cm = 10 cm
' m1V
From (i) and (ii) V= K1
m'–m  = K2 = 4K3  K1 = 4K2 = 16 K3 .
4
Torque of friction about point P is zero Angular
K 1.A.(10 ) K 2 A(20) K 2 A ( 40)
momentum will remain conserved about this point = 20 cm = 10 cm
40 cm
mR 2 mR 2 K1
 1  mVR  = K2 = 4K3  K1 = 4K2 = 16 K3 .
2 2 4

m'–m 2. For the elastic collision, total kinetic energy is


Solving this we get ' = .
3m'–m constant.
If we consider both the colliding bodies as system,
5. M1 is very large as compared to M 2. Hence for
then net work done on the system is zero (By work
collision between M1 and M2, M1 can be considered energy theorem)
equivalent to a wall and M2 as a small block. Thus K1 + K2 = K'1 + K'2 .... (1)
the velocity of M2 will be 2v o after collision with M1 K is kinetic energy of object before the collision
and K' is kinetic energy of object after the collision
. Similarly after collision between M 2 and M3, the
In general masses of both the objects are different,
velocity of M 3 will be 2(2v o). In sequence, the So speed of each object becomes different after
velocity of M4 shall be 2(2(2v o)) = 8 v o after collision collision K1  K'1 and K2  K'2
with M3. By work energy theorem
W 1 = K'1 – K1 .... (2)
W 2 = K'2 – K2 .... (3)
6. The component of velocity of charged particle along From (1), (2) and (3)
the magnetic field does not change. The component W 1  0, W 2  0
of velocity of charged particle normal to magnetic W1 + W 2 = 0
W1 = – W2
field only changes in direction but always remains
i.e. the work done by the first object on the second
normal to magnetic field. Hence angle between object is exactly the opposite of the work done by
velocity and magnetic field remains same. the second object on the first object.

3. A, B, D
qB 10 3 3800
7. f = =  4. The F.B.D. of cylinder is as shown. In equilibrium 
2m 19 2 T = Mg = tension in string
In equilibrium, net force on cylinder is zero
10 4  Torque is same about any axis.
=  Torque on cylinder about any point is zero.
 38
A = Mg 2R – mg sin R = 0

8. Pitch = T.V|| T
 
1 V.B  38 .400 4
= f.   4 = m f
| B | 10 . 3800 10 3


= m A
250
mgsin

DPPS FILE # 242

HAVE CONTROL ¾® HAVE PATIENCE ¾® HAVE CONFIDENCE Þ 100% SUCCESS


8. If R denotes radius of curvature of curved surface,
m sin 
M= Hence for only one value of M then from above figure
2
3 × (2R – 3) = 4 × 4
cylinder can remain in equilibrium.
 A is true, B is false 25
or R = mm
6
When the cylinder rolls up the incline, sense of
From the formulae of focal length for plano-convex
rotation of cylinder about center of mass is clockwise.
lens
Hence T > f.  C is false.
When the cylinder rolls down the incline, sense of
rotation of cylinder about center of mass is
anticlockwise. Hence T < f.  D is True.
4

5. When S2 is closed current in inductor 3 2R-3


4

R 25 25  3  1
 
f =  1 
6 3 1=
 ×

6 3  1  3  1  
remains, i =

2R 25
= ( 3  1) cm
12
  V1 +   V1  
 V 1 
2 
 =
3 
R 2R 2R  
DPP NO. - 76
 Potential difference
2 
(V) =  = Ans.
3 3 1.

di 2 di 2
And L = =+ Ans.
dt 3 dt 3L
6. From ray diagram it is clear that ray emerges out of
lens parallel to itself. Hence the angle of deviation
caused by the lens is 0°.

21  2 2 
i= =
R1  R 2 R1  R 2
60°
r r d
60° Where  = is the net emf in the circuit.
dt

R 2 – R1 
7. From snells law at first interface V1 – V2 = ( – iR1) – (  – iR2) = R1  R 2
sin 60 = 3 sinr or r = 30°
Since the emergent ray is parallel to initial incident 2. Mirror formula :
ray, the portion of lens used for refraction can be
assumed as slab 1 1 1
 
Hence lateral displacement v  280 20

sin(i  r ) sin(60  30 ) 1 1 1
= t 3 = 3 mm.  
cos r cos 30 v 20 280

DPPS FILE # 243

HAVE CONTROL ¾® HAVE PATIENCE ¾® HAVE CONFIDENCE Þ 100% SUCCESS


electric force is opposite to that of magnetic force.
1 14  1
 Direction of electric field between the plates is
v 280
opposite to that of direction of force on the negative
280 (electron) charge.
v=
15

2
v
v  = –   .v om
u

2
 280 
 v = –   .15
 15  280 

15 CM L/2
 v = 6. Hinge
15  15

1
v =  m/s Ans. L/2
15
Ma

CM
10m/s v1 v2 Mg
A B A B
3.

m × 10 = mv 1 + mv 2

 10 = v 1 + v 2 ...(i)
1 From the cart's frame
and × 10 = v 2 – v 1 ...(ii)
2 W all = KE2 – KE1
From  and 
L  L
5 15  Ma   + Mg    = 0 – 0
v1 = m/s ; v2 = m/s 2  2
2 2
 a=g
Distance between the two blocks
S = (– v 1 + v 2) . t

 5 15  N1
=    × 5 = 25 m
 2 2 
N2 Hinge
4. Heat obviously flows from higher temperature to lower
temperature in steady state.  A is true.
1
Temperature gradient  cross sec tion area in 7. Ma

steady state.  B is false. Mg


Thermal current through each cross section area is
same.  C is true.
Temperature decreases along the length of the rod
Initially rod is at rest
from higher temperature end to lower temperature end.
 D is false. So, N1 = Mg] N1 = Mg
5. A,B,C Torque = 
  2
Using – e ( V  B) for the region outside the plates,  L   ML 
Ma  2    3 

direction of magnetic field can be found. Inside the    
plates, net force on the electron is zero hence

DPPS FILE # 244

HAVE CONTROL ¾® HAVE PATIENCE ¾® HAVE CONFIDENCE Þ 100% SUCCESS


3. The direction of forces on the two elements taken
3  L 
 a=   symmetrical on two sides of the y–axis are shown.
4  2 
Clearly the net force will be on negative x-axis.
3 L
 g=
4 2

Ma + N2 = MaCM

3 
 Mg + N2 = M  g 
4 

Mg
 N2 = – 4. For a ball rolling without slipping on a fixed rough
4
surface, no work is done by friction.
17 Mg
N= N12  N22 =
4 5. p - Implse = Ft = 3mgt

8. Equation of motion for the cart 6. Let the angular speed of disc when the balls reach
Mg the end be . From conservation of angular
– + f = 2Ma
4 momentum

Mg 1 1 m 2 m 2
 f = 2Ma + mR2 0= mR2  + R + R  or 
4 2 2 2 2

9Mg 0
 f= =
4 3
7. The angular speed of the disc just after the balls leave
DPP NO. - 77 0
the disc is  =
3
1. It is clearly visible from all graphs that as x-
Let the speed of each ball just after they leave the
increases. Velocity changes sign. Since this is
disc be v.
not possible, no graph represents the possible
From conservation of energy
motion.
1 1 2
1 1  1 m 2
 mR  02 =  mR 2  2 +  v
2. Because of increase in magnetic field with time, 2 2  2 2  2 2
electric field is induced in the circular region and 1 m 2
represented by lines of forces as shown in figure. +  v
2 2
The signs of minimum work done by external agent
in taking unit positive charge from A to C via path solving we get
APC, AOC and AQC are 2R0
v=
3

NOTE : v = (R)2  v r2
; v r = radial velocity of the ball

1m
8. Workdone by all forces equal Kf – Ki =   v2 =
2 2 

mR 2 02
W APC = – ve, W AOC = 0, W AQC = + ve 9
 (C) is the correct choice.

DPPS FILE # 245

HAVE CONTROL ¾® HAVE PATIENCE ¾® HAVE CONFIDENCE Þ 100% SUCCESS


5. Since angular velocity is constant, acceleration of
DPP NO. - 78 centre of mass of disc is zero. Hence the
magnitude of acceleration of point S is 2x where
 is angular speed of disc and x is the distance of
S from centre. Therefore the graph is
1.

For the just completing the circular motion,


minimum velocity at bottom in

vB = 5gR
Energy conservation b/w point A and B Sol. 14 to 16
1 The free body diagram of plank and disc is
MgH + 0 = 0 + mv B2 Applying Newton's second law
2
F – f = Ma1 .... (1)
1 f = Ma2 .... (2)
MgH = m (5gR)
2 1
FR = MR2  .... (3)
2
5R
H=
2

  d
3. E . dr =–
dt
and take the sign of flux according to right hand
curl rule get.
 
E . d r = – (– (–A) – (–A) + (–A)) = –A
from equation 2 and 3
R
    a2 =
4. F = F     B 2
From constraint a1 = a2 + R

 = AD = R ( î  ĵ)  a1 = 3a2 .... (4)
 3F F
B = B0 ( î  ĵ  k̂ ) Solving we get a1 = and =
4M 2MR

 F = RB 0 ( î  ĵ) × ( î  ĵ  k̂ ) = RB 0 If sphere moves by x the plank moves by L + x.
The from equation (4)
î ĵ k̂ L
L + x = 3x or x =
1 1 0 2
= RB0 ( î  ĵ  2k̂ )
1 1 1

F = RB0 6
Aliter :

B  B0 ( î  ĵ  k̂ )

 = R ( î  ĵ)
 
B  =0
 Angle = 90°
 F = B = 3 B0  2 R= 6 B0 R

DPPS FILE # 246

HAVE CONTROL ¾® HAVE PATIENCE ¾® HAVE CONFIDENCE Þ 100% SUCCESS


Method II
DPP NO. - 79
1. FBD for sphere & block
k
m m
m fr /////////////////////////////////////////////////////////////////////////////////////
a1
m
fr k
m m
a2
/////////////////////////////////////////////////////////////////////////////////////////////////////////////////////////////////////

x1 x2
fr mg
a1 = =
m m  mx 1 = mx 2  x 1 = x 2
force equation for first block;
fr mg
a2 = =
m m 2k d2 x1
  (x 1 + x 2 ) = –m
a1  g î a 2  g î
3 dt 2
   Put x 1 = x 2
a rel  a1  a 2  2g î
arel = 2g. d2 x 1 4k
 + × x1 = 0
dt 2 3m
2. The electron ejected with maximum speed v max are
stopped by electric field E = 4N/C after travelling a 4k
 2 =
distance d = 1m 3m

1 3m
 mVmax2 = eE d = 4eV  T = 2
2 4K

1240
The energy of incident photon = = 6.2 eV
200
From equation of photo electric effect
M
1 5. =
mv max2 = h – 0 ( 4 / 3)R ( 4 / 3 )(R / 2) 3
3
2
 0 = 6.2 – 4 = 2.2 eV.
8 1
M1 = M , M2 = M
10 7 7
3. In steady state current from battery = = 5A
2
In parallel inductors L1I1 = L2I2 all the time 2  2  R 2 2
R  
= M 1 R2 –  M2    M2   
L2 3 5 5 2  2  

 i1 = L  L i = × 5 = 3A
1 2 32
57
;= MR2
4. Both the spring are in series 140
K(2K ) 2K
 Keq = =
K  2K 3
Time period P0

 m1m 2
T = 2 where  = H
K eq m1  m 2 2
6.
m
Here = V P0
2

m 3 3m
 T = 2  = 2 H
2 2K 4K V= 2gh = 2g   = gH
2

DPPS FILE # 247

HAVE CONTROL ¾® HAVE PATIENCE ¾® HAVE CONFIDENCE Þ 100% SUCCESS


3 1
= . mv 2
2 2
7.
K.E. of plank 2mv 2 8
 = 3 = .
K.E. of sphere mv 2 3
4

2
2. Ix = Iy = Iz = mR2
5

3. The spring is never compressed. Hence spring shall


Let h be height of water column just after putting
exert least force on the block when the block is at
cylinder,
topmost position.
 A  H
h´ A      A 
 3  2

3
 h´ = H
4

3
V´ = 2gh´ = gH
2

H A
8.     A   = h A 
2
  3 2
Fleast = kx0 – kA = mg – m2A = mg – 4 mA
T2
H
 h =
3
4. KEmax = (5 – ) eV
when these electrons are accelerated through 5V,
2
v = 2gh = gH they will reach the anode with maximum energy
3
= (5 –  + 5)eV
 10 –  = 8
 = 2eV Ans.
DPP NO. - 80 Current is less than saturation current because if
slowest electron also reached the plate it would have
1. Let velocity of c.m. of sphere be v. The velocity of 5eV energy at the anode, but there it is given that the
the plank = 2v. minimum energy is 6eV.
1
Kinetic energy of plank = × m × (2v)2 = 2mv 2 5. By principal of energy conservation.
2
PB = PR + PL
1 Near the starting of the circuit
Kinetic energy of cylinder = mv 2
2 di
PR = i 2 R and PL = L i .
dt
1  1 mR 2 2 
+ + 
2 2  di
As has greater value at the starting of the
dt
1  1 circuit, P L > PR
= mv 2 1  
2  2

DPPS FILE # 248

HAVE CONTROL ¾® HAVE PATIENCE ¾® HAVE CONFIDENCE Þ 100% SUCCESS


Sol. 1 to 3. : When connected with the DC source

12
R= =3
4

V
When connected to ac source  =
Z

12
 2.4 =  L = 0.08 H Net torque on rod about hinge 'O' = 0
3  2L2
2
L
Using P = rms Vrms cos   N1 × L = mg ×
2
2
Vrms
= cos  mg
Z or N1 
2
2
Vrms R Net torque on cylinder about its centre C is zero.
= 1 2 = 24 W
2
R  ( L – )  f 1R = f 2 R or f1  f2
C
Net torque on cylinder about hinge O is zero.
 N2 × L = N1 × L + mgL
DPP NO. - 81
3 mg
or N2 =
2. The potential difference across the inductor is e 2
= E– iR.
Hence the plot of e versus i is a straight line with
8. The magnetic force on bob does not produce any
negative slope.
restoring torque on bob about the hinge. Hence this
force has no effect on time period of oscillation.
Therefore both statements are correct and statement-
2 is the correct explanation.

3. Equation can be written as i = 2 sin 100  t + 2 sin


(100  t + 120º)
so phase difference  = 120º

= A 12  A 22  2A 1A 2 cos 

 1
= 4  4  2  2  2 –  = 2 so effective value will
 2

rms. value = 2 / 2 = 2A
nh h
4. Angular momentum = = ( n = 1)
2 2

h h
(mvr) = n. = (n = 1)
2 2
Sol.5 to 7.
FBD of rod and cylinder is as shown.

DPPS FILE # 249

HAVE CONTROL ¾® HAVE PATIENCE ¾® HAVE CONFIDENCE Þ 100% SUCCESS


 
DPP NO. - 82 8.  B  d  along any closed path within a uniform

1 magnetic field is always zero. Hence the closed


 i dt
1
  4
path can be chosen of any size, even very small
0 size enclosing a very small area. Hence we can
3. <i> =
1
=2 2  sin t  4  dt =
0
 prov e that net current through each area of
4. The hydrogen atom is in n = 5 state. infinitesimally small size within region of uniform
 Max. no of possible photons = 4 magnetic field is zero. Hence we can say no current
To emit photon in ultra violet region, it must jump to n (rather than no net current) flows through region of
= 1, because only Lyman series lies in u.v. region. uniform magnetic field. Hence statement -2 is correct
Once it jumps to n = 1 photon, it reaches to its ground explanation of statement-1.
state and no more photons can be emitted. So only
one photon in u. v. range can be emitted.
If H atom emits a photon and then another photon of
Balmer series, option D will be correct.

7. The FBD of cylinder is as shown

Resolving forces along x and y axis

x
y

fB= NB
NA NB

fA= NA mg

mg
NA + f B = .... (1)
2
f B =  NB
mg
NB – f A = .... (2)
2
f A =  NA
solving we get

 Mg(1  )
fA = and
2 (1   2 )

 Mg(1  )
fB =
2 (1   2 )
Angular acceleration

( f A  fB ) R 2 2 g
= =
MR 2 R(1   2 )
2

DPPS FILE # 250

HAVE CONTROL ¾® HAVE PATIENCE ¾® HAVE CONFIDENCE Þ 100% SUCCESS


IIT- JEE
MEDICAL
FOUNDATION(11 TH & 12TH)

PRE-FOUNDATION(8TH , 9TH, & 10TH )

HAVE CONTROL ¾® HAVE PATIENCE ¾® HAVE CONFIDENCE Þ 100% SUCCESS

You might also like